Vous êtes sur la page 1sur 73

Applied Quantum Mechanics

Chapter 1 Problems and Solutions

LAST NAME FIRST NAME

Useful constants MKS (SI)

c = 2.99792458 × 10 m s
8 –1
Speed of light in free space
h = 6.58211889 ( 26 ) × 10
–16
Planck’s constant eV s
h = 1.054571596 ( 82 ) × 10
– 34
J s
e = 1.602176462 ( 63 ) × 10
– 19
Electron charge C
m0 = 9.10938188 ( 72 ) × 10
– 31
Electron mass kg
mn = 1.67492716 ( 13 ) × 10
– 27
Neutron mass kg
Proton mass mp = 1.67262158 ( 13 ) × 10 – 27 kg
Boltzmann constant k B = 1.3806503 ( 24 ) × 10 –23 J K –1
k B = 8.617342 ( 15 ) × 10 –5 e V K– 1
Permittivity of free space ε 0 = 8.8541878 × 10 – 12 F m –1
Permeability of free space µ 0 = 4π × 10 – 7 H m – 1
Speed of light in free space c = 1 ⁄ ε0µ0
NA = 6.02214199 ( 79 ) × 10
23 –1
Avagadro’s number mol
a B = 0.52917721 ( 19 ) ×10
–10
Bohr radius m
4πε 0 h 2
a B = ----------------
-
m 0e2

Inverse fine-structure constant α –1 = 137.0359976 ( 50 )


4 πε 0 hc
α – 1 = -----------------
-
e2

Applied quantum mechanics 1


PROBLEM 1
A metal ball is buried in an ice cube that is in a bucket of water.
(a) If the ice cube with the metal ball is initially under water, what happens to the water level
when the ice melts?
(b) If the ice cube with the metal ball is initially floating in the water, what happens to the water
level when the ice melts?
(c) Explain how the Earth’s average sea level could have increased by at least 100 m compared
to about 20,000 years ago.
(d) Estimate the thickness and weight per unit area of the ice that melted in (c). You may wish
to use the fact that the density of ice is 920 kg m -3, today the land surface area of the Earth is about
148,300,000 km2 and water area is about 361,800,000 km2.

PROBLEM 2
Sketch and find the volume of the largest and smallest convex plug manufactured from a sphere
of radius r = 1 cm to fit exactly into a circular hole of radius r = 1 cm, an isosceles triangle with
base 2 cm and a height h = 1 cm, and a half circle radius r = 1 cm and base 2 cm.

PROBLEM 3
An initially stationary particle mass m1 is on a frictionless table surface and another particle
mass m2 is positioned vertically below the edge of the table. The distance from the particle mass
m1 to the edge of the table is l. The two particles are connected by a taught, light, inextensible
string of length L > l.
(a) How much time elapses before the particle mass m1 is launched off the edge of the table?
(b) What is the subsequent motion of the particles?
(c) How is your answer for (a) and (b) modified if the string has spring constant κ?

PROBLEM 4
The velocity of water waves in shallow water may be approximated as v = g h where g is the
acceleration due to gravity and h is the depth of the water. Sketch the lowest frequency standing
water wave in a 5 m long garden pond that is 0.9 m deep and estimate its frequency.

PROBLEM 5
(a) What is the dispersion relation of a wave whose group velocity is half the phase velocity?
(b) What is the dispersion relation of a wave whose group velocity is twice the phase velocity?
(c) What is the dispersion relation when the group velocity is four times the phase velocity?

PROBLEM 6
A stationary ground-based radar uses a continuous electromagnetic wave at 10 GHz frequency
to measure the speed of a passing airplane moving at a constant altitude and in a straight line at
1000 km hr-1. What is the maximum beat frequency between the out going and reflected radar
beams? Sketch how the beat frequency varies as a function of time. What happens to the beat fre-
quency if the airplane moves in an arc?

2
PROBLEM 7
How would Maxwell’s equations be modified if magnetic charge g (magnetic monopoles) were
discovered? Derive an expression for conservation of magnetic current and write down a general-
ized Lorentz force law that includes magnetic charge. Write Maxwell’s equations with magnetic
charge in terms of a field G = ε E + i µH .

PROBLEM 8
The capacitance of a small metal sphere in air is C0 = 1.1 × 10
–18
F . A thin dielectric film
with relative permittivity ε r 1 = 10 uniformly coats the sphere and the capacitance increases to
2.2 × 10
–18
F . What is the thickness of the dielectric film and what is the single electron charging
energy of the dielectric coated metal sphere?

PROBLEM 9
(a) A diatomic molecule has atoms with mass m1 and m2 . An isotopic form of the molecule has
atoms with mass m'1 and m'2 . Find the ratio of vibration oscillation frequency ω / ω' of the two
molecules.
12 16
(b) What is the ratio of vibrational frequencies for carbon monoxide isotope 12 ( C O ) and
13 16
carbon monoxide isotope 13 ( C O )?

PROBLEM 10
(a) Find the frequency of oscillation of the particle of mass m illustrated in the Fig. The particle
is only free to move along a line and is attached to a light spring whose other end is fixed at point A
located a distance l perpendicular to the line. A force F0 is required to extend the spring to length l.
(b) Part (a) describes a new type of child’s swing. If the child weighs 20 kg, the length l = 2.5
m, and the force F0 = 450 N, what is the period of oscillation?

Fixed
point A

Spring
Length, l

Mass, m

Displacement, -x

Applied quantum mechanics 3


SOLUTIONS
Solution 1
(a) The water level decreases. If ice has volume V then net change in volume of water in the
bucket is ∆ V = V ( ρice ⁄ ρ wate r – 1 ) .
(b) Again, the mass of the volume of liquid displaced equals the mass of the floating object.
Assuming the metal has a density greater than that of water, the water level decreases when the ice
melts.
(c) If there is just ice floating in the bucket, the water level does not change when the ice melts.
This fact combined with the results from part (a) and (b) allows us to conclude that only the ice
melting over land contributes to increasing the sea level.
(d) Today 71% water, 29% land, ratio is 2.45. The average thickness of ice on land is simply
100 m × 2.45 = 245 m. If one assumes half the land area under ice, then average thickness of ice is
490 m. If this is distributed uniformly from thin to thick ice, then one might expect maximum ice
thicknesses near 1000 m (i.e. ~ 3300 ft high mountains of ice). Ice weighs one metric tone per
cubic meter so the weight per unit area is the thickness in meters multiplied by tones.

Solution 2
We are asked to sketch and find the volume of the largest and smallest convex plug manufac-
tured from a sphere of radius r = 1 cm to fit exactly into a circular hole of radius r = 1 cm, an isos-
celes triangle with base 2 cm and a height h = 1 cm, and a half circle radius r = 1 cm and base 2 cm.
The minimum volume is 1.333 cm 3 corresponding to a geometry consisting of triangles placed
on a circular base as shown in the Fig.

2r = 2 cm

h = 1 cm

r = 1 cm

2 cm

To calculate this minimum volume of the plug consider the triangle at position x from the origin
has height k, base length 2l, and area kl.

4
h
r

r r
x
k

l l

(r - x)

l = k = ( r + x) ( r – x) = r – x
2 2 2 2

Volume of the plug is


r r r
x3 4
Vol = 2 ∫ k 2 dx = 2 ∫ ( r 2 – x 2 ) dx = 2 r 2 x – ---- = - r3
3 0 3
0 0
Since r = 1 cm the total volume is exactly 4/3 = 1.333 cm 3 .
To find the maximum volume of the plug manufactured from a sphere of radius r = 1 cm we
note that the geometry is a half sphere with two slices cut off as shown in the following Fig. The
geometry is found by passing the sphere along the three orthogonal directions x, y, and z and cutting
using a circle, a triangle, and a half circle, and a triangle respectively. As will be shown, the volume
is 1.608 cm3. This is the maximum convex volume of the plug manufactured from a sphere of
radius r = 1 cm.

2r = 2 cm

h = 1 cm

r = 1 cm

2 cm

To calculate the volume we first calculate the volume sliced off the half sphere as illustrated in
following Fig.

Applied quantum mechanics 5


l
(r - x) l

r (r - x)
x l
r
r x

r/21/2 r

l = (r + x ) (r – x ) = r – x
2 2 2

so that area of disk radius l at position x is


Area = π ( r – x )
2 2

The volume of the disk is the area multiplied by the disk thickness dx. The total volume is the
integral from x = r ⁄ 2 to x = r . Remembering to multiply by 2 because there are two slices
gives total volume sliced off
r r
x3
= 2πr 3   1 – --- –  ------- – ---------- 
1 1 1
Vol off = 2 π ∫ ( r 2 – x 2 ) dx = 2π r 2 x – ----
3 0
 3  2 6 2 
r⁄ 2

Vol off = 2 πr  - – ---------- 


3 2 5
 3 6 2
The volume of the plug is just the volume of a half sphere minus Vol off .
3
Vol = ----------- – 2 πr  -- – ----------  = 2πr  -- – -- + ----------  = 2πr  ---------- – ---
2πr 3 2 5 3 1 2 5 3 5 1
    
3 3 6 2 3 3 6 2 6 2 3
Since r = 1 cm the total volume is 1.608 cm3.
If we lift the restriction that the plug is manufactured from a sphere of radius 1 cm, then calcu-
lating the maximum convex volume turns out to be a bit complicated as may be seen in the follow-
ing Fig. The geometry is found by cutting along the three orthogonal directions x, y, and z using a
circle, a triangle, and a half circle, and a triangle respectively. The volume is 1.659 cm 3 .

2r = 2 cm

h = 1 cm

y
r = 1 cm
x

2 cm

6
Solution 3
(a) and (b). Force due to gravity acts on particle mass m2. This force is F = gm2 where g is the
acceleration due to gravity. The light string transmits the force to the second particle mass m1 that
is free to slide horizontally on the table surface. If x is the vertical position of the particle with mass
2
dx g m2 dx gm 2 t
m2 then 2
= ------------------
- and, starting from rest, speed = ------------------- with displacement
dt m 1 + m2 dt m 1 + m2

gm 2 t 2
x = --------------------------- . Eventually, particle m1 is launched with velocity v 1 from the end of the table, at
2 ( m 1 + m2 )
which point gravity causes it to accelerate in the x-direction under its own weight. If the distance of
2l ( m 1 + m 2 )
particle mass m1 to the edge of the table is l, then it takes time t = ----------------------------- for m1 to reach
m 2g
gm2 2l ( m 1 + m 2 )
the edge. The launch velocity is v 1 = ------------------- ----------------------------- . The initial angular momentum of
m1 + m2 m 2g
the system when particle mass m1 is launched from the edge of the table is Lv 1m1 m2 /(m1 +m2 ).
(c) If the string has a spring constant then part of the energy of the system can be stored in the
string during its trajectory. For example, this will happen during the initial acceleration phase.

Solution 4
We are given wave velocity, v = gh . This is a constant because h and g are fixed. Hence,
group velocity and phase velocity are the same and ω = 2 πf = gh × k = g h × 2 π ⁄ λ . For the
lowest frequency standing wave λ = 2 × 5 m and using g ~ 10, then the frequency of the wave is
f = 10 × 0.9 ⁄ 10 = 0.3 Hz .

Solution 5

(a) ω = Ak
0.5
.
∂ω dω ω
(b) Because the group velocity v g = is twice the phase velocity v p = ω ⁄ k we have = 2 ----
∂k dk k
dω dk
and so ------- = 2 ------ . Integrating both sides of this equation allows one to write
ω k
ln ω = 2 ln k + ln A = ln Ak 2 so that the dispersion relation is ω = Ak 2 where A is a constant.
(c) ω = Ak .
4

Solution 6
For a source moving at velocity v, the Doppler shifted frequency f’ is f′ = f 0 ⁄ ( 1 ± v ⁄ c ) where
f0 is the original frequency. The ± sign refers to the source moving towards or away from the detec-
tor. Hence, the maximum change in frequency relative to f0 is
∆ f = f 0 ( 1 – 1 ⁄ ( 1 ± v ⁄ c ) ) = ±f 0 ( v ⁄ ( c + v ) ) . In our case, v = 3.6 × 10 m s , c = 3 × 10 m s ,
8 -1 8 -1

and f0 = 10 10 Hz giving ∆f = 9.26 kHz. The component of velocity in the direction of the detector is
v × cos(θ) where θ is the angle between the ground and the plane flying at height h. A simple
expression for the time dependence of the angle θ = θ(t) is found knowing that the aircraft is mov-
ing at a constant velocity (and assuming the airplane flies right over the ground station). If the dis-

Applied quantum mechanics 7


2 2 2 2
tance to the plane (range) is l then l = h + v t (assuming we set t = 0 when the plane is
overhead). Then cos ( θ ( t ) ) = v t ⁄ l = v t ⁄ h +v t = 1⁄ h ⁄ v t + 1 . The Doppler shift as a
2 2 2 2 2 2

function of time is ∆f × cos(θ(t)). If the plane does not fly overhead it is necessary to introduce
another angle φ to describe its position.
If the plane is flying in an arc the Doppler shift can decrease. For example, it will be zero if the
plane is flying in a circle centered on the ground station. Assuming the plane flys at a constant
speed, whatever trajectory the plane describes, it cannot have a Doppler shift that is greater than the
maximum value we calculated ∆f for that velocity.
The reason why a typical radar antenna at an airport is highly directional and rotates is to find
the angular position of the aircraft with respect to the radar detector. Pulsed radar can be used to
find the range (distance) to the aircraft. The trajectory of the aircraft can be obtained by comparing
sequential position updates. Doppler radar is not required.

Solution 7
If magnetic charge g existed Maxwell’s equations would have to include magnetic charge den-
sity ρ m and magnetic current density J m as well as the usual electron charge density ρ e and elec-
tron current density J e . The new equations would be
∇ ⋅ D = ρe
∇ ⋅ B = ρm
∂B
∇×E = – ------- – Jm
∂t

∇×H = J e + ∂D
-------
∂t
where D = ε E and B = µ H .
Conservation of magnetic current J m = ρ m v is expressed as

∂ρm
+ ∇ ⋅ Jm = 0
∂t
For a particle carrying both electric charge e and magnetic charge g, the Lorentz force is
F = e( E + v × B ) + g( B – v × E )
Reminding ourselves of the SI units used for magnetic intensity H[A m-1], magnetic flux density
B[T] where B = µH, electric field strength E[V m -1 ],the displacement vector field D = εE, permia-
bility µ[N A-2] or [H m -1 ], permittivity ε[A 2 s2 N-1 m -2] or [F m-1 ], and ε 0 = 1 ⁄ µ 0 c where c is
2

the speed of light in free-space. If G G corresponds to energy density U = ( E ⋅ D + B ⋅ H ) ⁄ 2


*

ε µ
-- E + i --- H since ε E and µ H have dimensions of energy
2 2
then it makes sense to write G =
2 2
density U[J m -3]. Maxwell’s equations in terms of E and H with magnetic charge can be written
ε ρe
-- ∇ ⋅ E = ---------
-
2 2ε
µ ρm
--- ∇ ⋅ H = ----------
-
2 2µ
ε ε ∂H
- ∇×E = – --- µ ------- + Jm 
2 2 ∂t 

8
µ µ ∂E
--- ∇× H = ---  ε ------- + J e 
2 2  ∂t 
It follows that

ρe ρm
∇ ⋅ G = ---------- + i ----------
-
2ε 2µ
and
ε µ ε ∂H ε µ ∂E µ
∇ ×G = -- ∇× E + i --- ∇×H = – --- µ ------- – - J m + i --- ε ------- + i --- J e
2 2 2 ∂t 2 2 ∂t 2
which can be rewritten as

1 dG Jm iJ
---------- ∇ × G = i - + ---------e-
– ----------
εµ d t 2µ 2ε

since
dG µ ∂H ε∂E
i = – --- ------- + i -- -------
dt 2 ∂t 2 ∂t
The complex field G can be used to simplify the usual four Maxwell equations to just two equa-
tions.

Solution 8

We are given the information that a small metal sphere has capacitance C 0 = 1.1 × 10– 18 F in air.
First, we calculate the radius r 0 of the sphere using the formula
C 0 = 4 πε 0 r 0
It follows that
1.1 × 10
–18
C0
r 0 = ----------- = ----------------------------------------- = 10 n m
4π × 8.85 × 10
–12
4πε 0
Now consider the case when the metal sphere radius r0 with charge Q is coated with a dielectric of
relative permittivity εr 1 to radius r1 and ε r2 from r 1 radius to r2 . The voltage drop from r 0 through
the dielectrics to a much larger metal sphere of radius r2 is found by integrating
r0 r1 r0 r1
Q Q
V = – ∫ E rd r – ∫ E r dr = – ∫ ----------------------2 dr – ∫ ----------------------2 dr
r1 r r1 4πε 0 ε r 1 r
2 r 4πε 0 ε r 2 r 2

V = -----------  ---------- – ---------- + ---------- – ---------- 


Q 1 1 1 1
4πε 0  ε r1 r 0 ε r 1 r 1 ε r 2 r 1 ε r2 r 2
For this particular problem we now take the limit r 2 → ∞ . Capacitance
Q 4πε 0 4πε 0
C = ---- = ---------------------------------------------------------- = ------------------------------------------------
-
V  ---------- – ---------- + ---------- – 0 
1 1 1  1 1  ε r2 – ε r1 
εr r εr r ε r2 r 1   ---------- – ----  ----------------- 
1 0 1 1
 ε r1 r 0 r 1  ε r 1 ε r  2

C C  ε r2 – ε r1 
-  = 4 πε 0
---------- – ----  ----------------
ε r1 r 0 r 1  ε r1 ε r 
2

Applied quantum mechanics 9


C C  ε r 2 – ε r 1
---------- – 4πε 0 = ----  -----------------
ε r1 r 0 r 1  ε r1 ε r 
2

 ε r2 – ε r1  ε r1 r 0 ε r2 – ε r1 
=  ----------------
r0
r 1 = C  ----------------
-  --------------------------------

- --------------------------

ε
 r1 r 2  ε C – 4πε ε
0 r1 0 r ε r2 ε r1 C 0 
 1 – -----------
 -
C 
Putting in the numbers. We have ε r1 = 10 , εr 2 = 1 , C0 = 1.1 × 10 –18 F , C = 2.2 × 10 –18 F ,
and r 0 = 10 nm , so that

r 1 =  ---------------  -------------------- = -- r 0 = 22.5 nm


1 – 10 r0 9
1  1 – 10 4
 ------ 
2
and we may conclude that the thickness of the dielectric film is r 1 – r 0 = 12.5 n m .
Charging energy ∆E = e ⁄ 2 C = 1.6 × 10 ⁄ 4.4 × 10
2 – 19 – 18
= 36 m eV . This value is similar to
ambient thermal energy k B T = 25 m eV .

Solution 9
(a) The molecule consists of two particles mass m 1 and mass m 2 with position r 1 and r 2
respectively. The center of mass coordinate is R and relative position vector is r . We assume that
the potential depends only on the difference vector r = r 2 – r 1 and if the origin is the center of
mass then m1 r 1 + m 2 r 2 = 0 , so that
m2
r 1 = -----------------------r
( m1 + m 2 )
and
m1
r 2 = -----------------------r
( m1 + m 2 )
Since, for example,
–m –m
r 1 = ---------2 r 2 = ---------2 ( r 1 – r )
m1 m1

r 1  1 + ------2  = ------2 r
m m
 m 1 m1
r 1 ( m 1 + m2 ) = m 2 r
m2
r 1 = ----------------------- r
( m1 + m 2 )
Now, combining center of mass motion and relative motion, the Hamiltonian is the sum of kinetic
and potential energy terms
· 2 1  · · 2
H = T + V = - m 1  R – -----------------------r - r + V ( r )
1 · m2 m1
+ -- m 2 R + ----------------------
2  (m 1 + m 2 )  2  ( m 1 + m2 ) 
where the total kinetic energy is
1 ·2 m1 m 22 · 2 2
m 1m 2 · 2 1 · 2 1 m 1 m2 · 2
T = -- ( m 1 + m2 )R + -------------------------- r + -------------------------- r = -- ( m1 + m 2 )R + -- -----------------------r
2 ( m1 + m 2 )
2
( m 1 + m2 )
2 2 2 ( m1 + m 2 )
or
1 · 2 1 ·2
T = -- M R + -- mr
2 2

10
where
M = m 1 + m2
and the reduced mass is
m 1 m2
m = -----------------------
( m1 + m 2 )

m1
r
m2

Because the potential energy of the two interacting particles depends only on the separation
between them, V = V ( r 2 – r 1 ) = V ( r ) . The frequency of oscillation of the molecule is deter-
mined by the potential and given by ω = κ ⁄ m where κ is the spring constant and m is the
reduced mass 1 ⁄ m = 1 ⁄ m 1 + 1 ⁄ m2 . Since the isotope form of the molecule interacts in the same
way, κ = κ′ , and the ratio of oscillation frequency is given by

ω′ m1 m 2 ( m′1 + m′ 2 )
----- = -----------------------------------------
ω m′1 m′ 2 ( m 1 + m 2 )
We could get the same result quite simply if we assume motion is restricted to one dimension.
In that case the force on mass m1 and mass m2 is given by their relative displacement multiplied by
the spring constant κ.

κ
m1 m2

u v

2
du
m1 = κ( v – u)
dt 2
2
dv
m2 2 = κ ( u – v )
dt
–i ωt
which has solution of the form e giving
( κ – m 1 ω )u – κ v = 0
2

– κu + ( κ – m2 ω 2 ) v = 0
giving a characteristic equation

Applied quantum mechanics 11


κ – m1 ω –κ
2
= ( κ – m 1 ω 2 ) (κ – m 2 ω 2 ) – κ 2 = 0
–κ κ – m 2ω
2

κ =  -------------------  ω = mω
m 1 m2 2 2

m1 + m2
and

κ  ------------------- =
m1 + m 2
ω = κ⁄m
m 1 m2
and, as before, since the isotope form of the molecule interacts in the same way, κ = κ′ , and
the ratio of oscillation frequency is given by

ω′ m1 m 2 ( m′1 + m′ 2 )
----- = ----------------------------------------
-
ω m′1 m′ 2 ( m1 + m 2 )
(b) To find the ratio of vibrational frequencies we use the result in part (a) and put in the atomic
ω 12C 13 × 16 × ( 12 + 16 ) 12 16
masses to give --------- = ----------------------------------------------- = 1.02 . As expected, the lighter C O molecule
ω 13C 12 × 16 × ( 13 + 16 )
13 16
vibrates at a higher frequency (by about 2%) than the C O molecule.

Solution 10
(a) For small displacement x we assume F0 doesn’t change much so for extension ∆l the work
done is F0 ∆ l . Hence, the potential energy of the spring is equal to the force F0 multiplied by the
extension ∆l . For x « l we have ∆l = l + x – l = x ⁄ 2l , so that the potential energy is
2 2 2

V = F0 x ⁄ 2l = κx ⁄ 2 . This is the potential energy of a harmonic oscillator with spring constant


2 2

κ = F0 ⁄ l . This oscillator has frequency ω = κ⁄m = F 0 ⁄ ml .


(b) The frequency of oscillation in radians per second is given by
ω = F0 ⁄ ml = 450 ⁄ 20 × 2.5 =
–1
9 = 3 rad s , so the period of oscillation is
τ = 1 ⁄ f = 2π ⁄ ω = 2.09 s , i.e., about 2 seconds.

12
Applied Quantum Mechanics

Chapter 2 Problems and Solutions

LAST NAME FIRST NAME

Useful constants MKS (SI)

c = 2.99792458 × 10 m s
8 –1
Speed of light in free space
h = 6.58211889 ( 26 ) × 10
– 16
Planck’s constant eV s
h = 1.054571596 ( 82 ) × 10
–34
J s
e = 1.602176462 ( 63 ) × 10
– 19
Electron charge C
m 0 = 9.10938188 ( 72 ) × 10
–31
Electron mass kg
m n = 1.67492716 ( 13 ) × 10
–27
Neutron mass kg
Proton mass m p = 1.67262158 ( 13 ) × 10 –27 k g
Boltzmann constant k B = 1.3806503 ( 24 ) × 10 –23 J K –1
k B = 8.617342 ( 15 ) × 10 – 5 eV K –1
Permittivity of free space ε 0 = 8.8541878 × 10 – 12 F m –1
Permeability of free space µ 0 = 4π × 10 –7 H m –1
Speed of light in free space c = 1 ⁄ ε0µ0
N A = 6.02214199 ( 79 ) × 10
23 –1
Avagadro’s number mol
a B = 0.52917721 ( 19 ) ×10
–10
Bohr radius m
4πε 0 h 2
a B = ----------------
-
m 0e2

Inverse fine-structure constant α –1 = 137.0359976 ( 50 )


4 πε 0 hc
α – 1 = -----------------
-
e2

Applied quantum mechanics 1


PROBLEM 1
(a) The Sun has a surface temperature of 5800 K and an average radius 6.96 × 108 m. Assuming
the mean Sun-Mars distance is 2.28 × 1011 m, what is the total radiative power per unit area inci-
dent on the upper Mars atmosphere facing the Sun?
(b) If the surface temperature of the Sun was 6800 K, by how much would the total radiative
power per unit area incident on Mars increase?

PROBLEM 2
If a photon of energy 2 eV is reflected from a metal mirror, how much momentum is
exchanged? Why can the reflection not be modeled as a collision of the photon with a single elec-
tron in the metal?

PROBLEM 3
Consider a lithium atom (Li) with two electrons missing.
(a) Draw an energy level diagram for the Li++ ion.
(b) Derive the expression for the energy (in eV) and wavelength (in nm) of emitted light from
transitions between energy levels.
(c) Calculate the three longest wavelengths (in nm) for transitions terminating at n = 2 .
(d) If the lithium ion were embedded in a dielectric with relative permittivity εr = 10, what
would be the expression for the energy (in eV) and wavelength (in nm) of emitted light from transi-
tions between energy levels.

PROBLEM 4
(a) There is a full symmetry between the position operator and the momentum operator. They
form a conjugate pair. In real-space momentum is a differential operator. Show that in k-space

position is a differential operator, ih , by evaluating expectation value
∂px

〈 x̂ 〉 = ∫ dxψ ( x )x ψ ( x )
*

–∞

in terms of φ(k x) which is the Fourier transform of ψ(x).


(b) The wave function for a particle in real-space is ψ ( x , t ) . Usually it is assumed that position
x and time t are continuous and smoothly varying. Given that particle energy is quantized such that

E = hω , show that the energy operator for the wave function ψ ( x, t ) is ih .
∂t

PROBLEM 5
A simple model of a heterostructure diode predicts that current increases exponentially with
increasing forward voltage bias. Under what conditions will this predicted behavior fail?

PROBLEM 6
Write down the Hamiltonian operator for (a) a one-dimensional simple harmonic oscillator, (b)
a Helium atom, (c) a Hydrogen molecule, (d) a molecule with nn nuclei and ne electrons.

2
PROBLEM 7
Calculate the classical velocity of the electron in the n-th orbit of a Li++ ion. If this electron is
described as a wave packet and its position is known to an accuracy of ∆x = 1 pm, calculate the
characteristic time ∆τ∆x for the width of the wave packet to double. Compare ∆τ∆x with the time to
complete one classical orbit.

PROBLEM 8
What is the Bohr radius for an electron with effective electron mass m* = 0.021 × m 0 in a
medium with low-frequency relative permittivity ε r 0 = 14.55 corresponding to the conduction
band properties of single crystal InAs?

PROBLEM 9
Because electromagnetic radiation possesses momentum it can exert a force. If completley
absorbed by matter, the absorbed electromagnetic radiation energy per unit time per unit area is a
pressure called radiation pressure.
(a) If the maximum radiative power per unit area incident on the upper Earth atmosphere facing
the Sun is 5.5 kW m -2 , what is the corresponding radiation pressure?
(b) Estimate the photon flux needed to create the pressure in (a).
(c) Compare the result in (a) with the pressure due to one atmosphere.

Applied quantum mechanics 3


SOLUTIONS

Solution 1
(a) 2.4 kW m -2.
(b) 4.5 kW m -2.

Solution 2
Assume the photon of energy E = 2 eV is incident from free-space at angle θ normal to a flat
metal mirror. The magnitude of photon momentum is p p h = hk = hω ⁄ c where frequency ω is
given by the quanta of photon energy E = hω . The momentum change upon reflection is
∆ p = ( 2hω ⁄ c ) cos θ which, for θ = 0, gives a value
2 E ⁄ c = 2 × 2 × 1.6 × 10 ⁄ 3 × 10 = 2.1 × 10
–19 8 –27 –1
k g m s . The reason why this reflection can
not be modeled as a collision of the photon with a single electron in the metal is that in such a situ-
ation it is not possible to conserve both energy and momentum and have the photon maintain its
wavelength. Collision with an electron would take away energy from the photon and cause a
change in photon wavelength. Since, by definition, reflected light has the same wavelength as the
incident light, photon energy cannot change.
Another way to express this is that the difference in dispersion relation for free electrons and
photons is such that light cannot couple directly to a free electron.

Solution 3
2
Z
E n = Ry -----2
n
122.4
For Li++ Z = 3 and so E n = -------------
2
eV .
n
2πh c – 10.1
Emission wavelength is λ = -------------------- = ---------------- nm , giving λ 3, 2 = 72.7 nm , λ 4, 2 = 53.8 n m ,
En2 – En1 1 1
--- – -----
n2 n1
λ 5, 2 = 48.1 n m .
2 4
– m0 Z e  1 1 
For ε r = 10 we have En 2 – En 1 = ---------------------------- --- – ----- , so divide energy differences by
( 4π ε0 ε r ) 2 h  n 2 n 1 
2

ε r = 100
2

Solution 4
(a) The expectation value of the position operator is
∞ ∞ ∞ ∞
1
〈 x̂ 〉 = ∫ dxψ ( x )x ψ ( x ) = ------ ∫ dx ∫ dk ′φ ( k′ ) e x ∫ d kφ ( k ) e
* * –i k′x ikx

–∞
2π –∞ –∞ –∞

∞ ∞ ∞
∂ e
ikx
〈 x̂ 〉 = ------ ∫ dx ∫ d k′φ ( k ′ )e x ∫ dkφ ( k )  ------- 
1 * – ik ′x

2π – ∞ – ∞ –∞
∂ k ix

4
∞ ∞ ikx ∞ ∞
∂
i kx
〈 x̂ 〉 = ------ ∫ d x ∫ d k′φ * ( k′ )e – ik′ x x  φ ( k ) ------- e 
1 e
2 π – ∞ –∞  ix –∞
– ∫ dk ∂ k φ ( k )  -------
ix 
–∞

∞ ∞ ∞ ∞ ∞ ∞
∂ ∂
∫ dk  ∂ k φ ( k )  e = ------
–1 i i( k – k′ )x
〈 x̂ 〉 = -------- ∫ d x ∫ dk ′φ ( k′ )e ∫ dxe ∫ ∫ dk d k′φ ( k ′ ) ∂ k φ ( k )
* – ik′ x i kx *

2πi –∞ –∞ –∞
2π –∞ – ∞ –∞

∞ ∞ ∞ ∞
∂ ∂ ∂
〈 x̂〉 = i ∫ ∫ d k dk ′φ ( k′ ) φ ( k )δ ( k – k ′ ) = i ∫ d kφ ( k′ ) φ ( k ) = ih ∫ d kφ ( k′ ) φ ( k )
* * *

–∞–∞
∂k –∞
∂ k –∞
∂ p

Hence, in k-space position is a differential operator, ih .
∂p
(b) The expectation value of the energy operator is
∞ ∞ ∞ ∞
1 –i ωt ′ i ωt
〈 Ê〉 = ∫ d ωψ ( ω )hω ψ ( ω ) = ------ ∫ d ω ∫ d t′φ ( t′ ) e h ω ∫ d tφ ( t) e
* *

–∞
2π –∞ –∞ –∞

∞ ∞ ∞
∂ e
iω t′
〈 Ê 〉 = ------ ∫ d ω ∫ d t′φ ( t′ ) e h ω ∫ d tφ ( t )  --------- 
1 * –i ωt ′

2 π–∞ –∞ –∞
∂ t iω
∞ ∞ ∞ ∞
ei ωt ∂ e iω t
〈 Ê〉 = ------ ∫ dω ∫ dt′φ * ( t′ )e –iω t ′ hω  φ ( t ) ------- ∫ d t ∂ t φ ( t )  ------- 
1

2π –∞ –∞  iω –∞ iω 
–∞

∞ ∞ ∞ ∞ ∞ ∞
–h – iω t ′ ∂  i ωt ih i ω( t – t ′ ) ∂
〈 Ê 〉 = -------- ∫ dω dt′φ ( t′ )e ∫ dt  ∂ t φ ( t )  e = ------ ∫ d ωe ∫ ∫ d td t′φ ( t′ ) ∂ t φ ( t )
* *
2 πi ∫ 2 π
–∞ –∞ –∞ –∞ –∞ – ∞

∞ ∞ ∞ ∞
∂ ∂ ∂
〈 Ê〉 = ih dt d t′ φ ( t′ ) φ ( t )δ ( t – t′ ) = ih ∫ d tφ ( t′ ) φ ( t ) = ih ∫ d tφ ( t′ ) φ ( t )
* * *
∫ ∫ ∂t ∂ t ∂ t
–∞–∞ –∞ –∞


Hence, in t-space energy is a differential operator, ih .
∂t

Solution 5
Series resistance. Space charging at current density j > nev . For bipolar devices with direct a
band gap can have stimulated emission.

Solution 6
We are asked to write down the Hamiltonian operator for (a) a one-dimensional simple har-
monic oscillator, (b) a Helium atom, (c) a Hydrogen molecule, (d) a molecule with nn nuclei and ne
electrons.
(a) The Hamiltonian for a particle mass m restricted to harmonic oscillatory motion of fre-
quency ω in the x-direction is

–h d
2 2 2
mω 2
H = T + V = -------- 2 + ---------- x
2m d x 2
(b) and (c) may be found from the general solution (d) for a molecule with n n nuclei and ne elec-
trons in which the Hamiltonian is
j = nn i = ne
–h –h
2 2
H = ∑ --------- ∇ 2 +
2M
∑ --------- ∇ 2 + V ( r )
2 m0
j=1 j i=1

Applied quantum mechanics 5


Here, the first term is the contribution to the kinetic energy from the n n nuclei of mass M j, the sec-
ond term is the kinetic energy contribution from the n e electrons of mass m0 , and V(r) is the poten-
tial energy given by
2 2
e2 ZZ e Ze
∑ ------------------ ∑ ------------------ – ∑ ----------------
j j′
V (r ) = + j
4πε r
i > i′ 0 i i′ 4 πε r
j > j′ 0 jj ′ 4 πε r
i, j 0 ij

The first term in the potential is the electron-electron coulomb repulsion between the electrons i and
i'. The second term is the nucleus-nucleus coulomb repulsion between the nuclei j and j' with
charges Zj and Z j' respectively. The third term is the coulomb attraction between electron i and
nucleus j.

Solution 7
For n = 1, orbit time ~ 10 -17 s, dispersion time ~ 10 -20 s.

Solution 8

4πε 0 ε r h 2 εr
a *B = ---------------------
2
= a B -------- = 0.529 × 10 –10 × 693 = 36.7 nm
m 0 m eff e me ff

Solution 9
(a) The maximum radiative power per unit area incident on the upper Earth atmosphere facing
the Sun is Stotal = 5.5 kW m-2 . The radiation pressure us just S total ⁄ c = 1.8 × 10 –5 N m– 2 .
(b) From Exercise 1 Chapter 2, hω average = 1.92 eV so the number of photons per second per
unit area is S total ⁄ e hω averag e = 1.8 × 10 22 s –1 m –2 .
(c) The radiation pressure in (a) is a small value compared to one atmosphere which is about
10 N m –2 . Never-the-less, electromagnetic radiation from the Sun can be absorbed, exert a force,
5

and change the direction of small particles in space. For example, it is responsible for continuously
sweeping dust particles out of the solar system. As another example, uncharged dust particles from
a comet can form a dust tail whose direction and shape is determined by electromagnetic radiation
pressure from the Sun. Interestingly, comets can also shed charged dust particles to form what is
called a gas tail. The gas tail is swept along by a stream of charged particles and magnetic field
lines emitted by the Sun that is called the solar wind. Often a comet will have two separate tails
because the solar wind often does not point radially outward from the Sun.

6
Applied Quantum Mechanics

Chapter 3 Problems and Solutions

LAST NAME FIRST NAME

Useful constants MKS (SI)

c = 2.99792458 × 10 m s
8 –1
Speed of light in free space
h = 6.58211889 ( 26 ) × 10
– 16
Planck’s constant eV s
h = 1.054571596 ( 82 ) × 10
–34
J s
e = 1.602176462 ( 63 ) × 10
– 19
Electron charge C
m 0 = 9.10938188 ( 72 ) × 10
–31
Electron mass kg
m n = 1.67492716 ( 13 ) × 10
–27
Neutron mass kg
Proton mass m p = 1.67262158 ( 13 ) × 10 –27 k g
Boltzmann constant k B = 1.3806503 ( 24 ) × 10 –23 J K –1
k B = 8.617342 ( 15 ) × 10 – 5 eV K –1
Permittivity of free space ε 0 = 8.8541878 × 10 – 12 F m –1
Permeability of free space µ 0 = 4π × 10 –7 H m –1
Speed of light in free space c = 1 ⁄ ε0µ0
N A = 6.02214199 ( 79 ) × 10
23 –1
Avagadro’s number mol
a B = 0.52917721 ( 19 ) ×10
–10
Bohr radius m
4πε 0 h 2
a B = ----------------
-
m 0e2

Inverse fine-structure constant α –1 = 137.0359976 ( 50 )


4 πε 0 hc
α – 1 = -----------------
-
e2

Applied quantum mechanics 1


PROBLEM 1
Prove that particle flux (current) is zero if the one-dimensional exponential decaying wave func-
– κ x – i ωt
tion in tunnel barrier of energy V0 and finite thickness L is ψ ( x, t ) = Be , where κ is a real
positive number and particle energy E = h ω < V0 .

PROBLEM 2
(a) Use a Taylor expansion to show that the second derivative of a wavefunction ψ ( x ) sampled
at positions x j = jh 0 , where j is an integer and h 0 is a small fixed increment in distance x, may be
approximated as

ψ ( x j – 1 ) – 2ψ ( x j ) + ψ ( x j + 1 )
2
d
ψ ( x j ) = -----------------------------------------------------------------
2 2
-
dx h0
(b) By keeping additional terms in the expansion, show that a more accurate approximation of
the second derivative is

– ψ ( x j – 2 ) + 16 ψ ( x j – 1 ) – 30ψ ( x j ) + 16ψ ( x j + 1 ) – ψ ( x j + 2 )
2
d
2
ψ ( x j ) = --------------------------------------------------------------------------------------------------------------------------------
2
-------
dx 12h 0

PROBLEM 3
Using the method outlined in Exercise 7 of Chapter 3 as a starting point, calculate numerically
the first four energy eigenvalues and eigenfunctions for an electron with effective mass
m *e = 0.07 × m0 confined to a potential well V(x) = V0 of width L = 10 nm with periodic boundary
conditions.
Periodic boundary conditions require that the wave function at position x = 0 is connected
(wrapped around) to position x = L. The wave function and its first derivative are continuous and
smooth at this connection.
Your solution should include plots of the eigenfunctions and a listing of the computer program
you used to calculate the eigenfunctions and eigenvalues.

PROBLEM 4
Using the method outlined in Exercise 7 of Chapter 3 as a starting point, calculate numerically
the first six energy eigenvalues and eigenfunctions for an electron with effective mass
m *e = 0.07 × m0 confined to a triangular potential well of width L = 20 nm bounded by barriers of
infinite energy at x < 0 and x > L. The triangular potential well as a function of distance x is given
by V(x) = V0 × x / L where V0 = 1 eV.
Explain the change in shape of the wave function with increasing eigenenergy.
Your solution should include plots of the eigenfunctions and a listing of the computer program
you used to calculate the eigenfunctions and eigenvalues.

PROBLEM 5
Calculate the transmission and reflection coefficient for an electron of energy E, moving from
left to right, impinging normal to the plane of a semiconductor heterojunction potential barrier of
energy V 0 , where the effective electron mass on the left-hand side is m 1 and the effective electron
mass on the right-hand side is m 2 .
If the potential barrier energy is V0 = 1.5 eV and the ratio of effective electron mass on either
side of the heterointerface is m1 / m2 = 3, at what particle energy is the transmission coefficient
unity? What is the transmission coefficient in the limit E → ∞ ?

2
SOLUTIONS

Solution 1
To prove that particle flux (current) is zero if the one-dimensional exponential decaying wave
– κx – iω t
function in tunnel barrier of energy V0 and finite thickness L is ψ ( x , t ) = Be , where κ is a
real positive number and particle energy E = hω < V 0 , we substitute the wave function into the
current operator

J = ----------- ψ * ( x , t ) ψ ( x, t ) – ψ ( x, t ) ψ * ( x , t ) 
– i eh d d
2m  dx dx 
– i eh
J = ----------- ( B* e – κx + i ωt ( – κB e – κx – i ωt ) – Be – κ x – i ωt ( – κB * e – κx + i ω t ) )
2m
κi eh 2 –2κx
J = ----------- B ( e –e ) = 0
– 2κ x

2m

Solution 2
(a) Consider the Taylor expansion for the function f(x).
f′′ ( x ) 2 f′′′ ( x ) 3 1 (n )
f ( x j + 1 ) = f ( x j ) + f′ ( x j ) h 0 + --------------j h 0 + ----------------j h 0 + … = ∑ -n!
-- ⋅ f ( xj ) ⋅ h0
n
2! 3! n

where j is an integer and h0 is a small fixed increment in distance x. Keeping terms to order h 0 , we
see that the first derivative is
f( x j + 1 ) – f ( x j )
f′ ( x j ) = --------------------------------
-
h0
Hence, the first derivative of a wavefunction ψ ( x ) sampled at positions x j = j h 0 may be approxi-
mated as
d ψ (x j + 1 ) – ψ (x j )
ψ ( x j ) = -------------------------------------
-
dx h0
2
To find the second derivative we keep terms to order h 0 so that
f′′ ( x ) 2
f ( x j + 1 ) = f ( x j ) + f′ ( x j ) h 0 + --------------j h 0
2
and
f′′ ( x ) 2
f ( x j – 1 ) = f ( x j ) – f′ ( x j ) h 0 + --------------j h 0
2
Adding these two equations gives
f ( x j – 1 ) + f ( x j + 1 ) = 2f ( x j ) + f′′ ( x j ) h 0
2

or
f ( x j – 1 ) – 2 f( x j ) + f ( x j + 1 )
f′′ ( x j ) = ----------------------------------------------------------
2
-
h0
so that the second derivative of a wavefunction ψ ( x ) sampled at positions x j = j h 0 may be
approximated as
ψ ( x j – 1 ) – 2ψ ( x j ) + ψ ( x j + 1 )
2
d
2
ψ ( x j ) = -----------------------------------------------------------------
-
dx h 20
This is the three-point approximation to the second derivative accurate to second order, 0 ( h 0 ) .
2

(b) By keeping five terms in the expansion instead of three one may obtain a more accurate
approximation of the second derivative accurate to fourth order, 0 ( h 0 ) . To see how, we use the fol-
4

lowing Taylor expansions with derivatives up to fourth-order.

Applied quantum mechanics 3


4 2
f ( x j + 2 ) = f ( x j ) + 2f′ ( x j ) h 0 + 2f′ ′ ( x j ) h 0 + -- f′′′ ( x j ) h 0 + - f′′′′ ( x j ) h 0
2 3 4
3 3
1 1 1
f ( x j + 1 ) = f ( x j ) + f′ ( x j ) h0 + - f′′ ( x j )h 0 + -- f′′′ ( x j )h 0 + ------ f′′′′ ( x j )h 0
2 3 4

2 6 24
1 1 1
f ( x j – 1 ) = f ( x j ) – f′ ( x j ) h 0 + -- f′′ ( x j ) h0 – -- f′′′ ( x j )h 0 + ------ f′′′′ ( x j ) h0
2 3 4
2 6 24
4 2
f ( x j – 2 ) = f ( x j ) – 2f′ ( x j )h 0 + 2f′′ ( x j )h 0 – -- f′ ′ ′ ( x j ) h 0 + -- f′′′′ ( x j ) h 0
2 3 4

3 3
To eliminate the terms in the first, third, and fourth derivative we multiply each equation by coeffi-
cients a, b, c, d, respectively, to obtain

a f ( x j + 2 ) + bf ( x j + 1 ) + c f ( x j – 1 ) + d f ( x j – 2 ) = ( a + b + c + d ) f ( x j ) + ( 2a + b – c – 2 d ) f′ ( x j ) h 0 +

 b c  +  ------ + - – --- – ------ f′′′ ( x j ) h 0 +  ------ + ------ + ------ + ------  f′′′′ ( x j )h 0


4a b c 4d 2a b c 2d
 2a + -2 + -2- + 2d f′′ ( x j )h 0  3 6 6 3 
2 3 4
 3 24 24 3 
The coefficients that eliminate terms in the first, third, and fourth derivative must satisfy
0 = ( 2 a + b – c – 2d )

0 =  ------ + - – --- – ------ 


4a b c 4d
3 6 6 3

0 =  2a ------ 
------ + ---b--- + ---c--- + 2d
 3 24 24 3 
The solution to this set of linear equations is
a = d
b = – 16d
c = – 16d
Setting a = 1 we obtain
f ( x j + 2 ) – 16f ( x j + 1 ) – 16f ( x j – 1 ) + f ( x j – 2 ) = –30 f ( x j) – 12f′′ ( x j )h 0
2

so that
– f ( x j + 2 ) + 16 f ( x j + 1 ) – 30f ( x j ) + 16 f ( x j – 1 ) – f ( x j – 2 )
f′′ ( x j ) = --------------------------------------------------------------------------------------------------------------------------
-
12h 20
so that the second derivative of a wavefunction ψ ( x ) sampled at positions x j = jh 0 may be
approximated as
– ψ ( x j – 2 ) + 16 ψ ( x j – 1 ) – 30ψ ( x j ) + 16ψ ( x j + 1 ) – ψ ( x j + 2 )
2
d
2
ψ ( x j ) = --------------------------------------------------------------------------------------------------------------------------------
2
-------
dx 12h 0

Solution 3
To find numerically the first four energy eigenvalues and eigenfunctions for an electron with
effective mass me = 0.07 × m 0 confined to a potential well V(x) = V0 of width L = 10 nm with
*

periodic boundary conditions we descretize the wavefunction and potential appearing in the time-
independent Schrödinger equation
h2 d
2
Hψ n ( x ) =  – ------- 2 + V ( x )  ψ n ( x ) = E n ψ n ( x )
2m d x
using a discrete set of N + 1 equally-spaced points such that position x j = j × h 0 where the index
j = 0, 1 , 2 …N , and h 0 is the interval between adjacent sampling points so one may define
x j + 1 ≡ x j + h 0 . The region in which we wish to solve the Schrödinger equation is of length
L = Nh 0 . At each sampling point the wave function has value ψ j = ψ ( x j ) and the potential is

4
V j = V ( x j ) . The second derivative of the discretized wave function we use the three-point finite-
difference approximation which gives

ψ ( x j – 1 ) – 2ψ ( x j ) + ψ ( x j + 1 )
2
d
2
ψ ( x j ) = -----------------------------------------------------------------
2
-
dx h0
Substitution into the Schrödinger equation gives a matrix equation
Hψ ( x j ) = – u j ψ ( x j – 1 ) + d ψ ( x j ) – u j + 1 ψ ( x j + 1 ) = Eψ ( x j )
where the Hamiltonian is a symmetric tri-diagonal matrix. The diagonal matrix elements are

h
2
d j = ---------2 + Vj
mh 0
and the adjacent off-diagonal matrix elements are

h
2
u j = -------------2
2mh 0
As usual, the wavefunction must be continuous and smooth. In addition, the periodic boundary
conditions require ψ ( x ) = ψ ( x + L ) so that ψ 0 ( x 0 ) = ψ N( x N ) . In this situation the Hamiltonian
becomes a N × N matrix H and the Schrödinger equation is

( d1 – E ) –u2 0 0 . . . –u1 ψ1
–u2 ( d2 – E ) –u3 0 . . . . ψ2
0 –u3 ( d3 – E ) –u 4 . . . . ψ3
0 0 –u4 ( d4 – E ) . . . . . ψ4
( H – EI )ψ = = 0
. . . . . . . . .
. . . . . . –uN – 2 0 ψN – 2
. . . . . . ( dN – 1 – E ) –uN – 1 ψN – 1
–u1 . . . . . – uN ( dN – E) ψN
where I the identity matrix. The matrix elements in the upper right and lower left corners are due to
the periodic boundary conditions. For the case we are interested in, the first four eigenenergies are:
E1 = 0 eV, E2 = 0.215 eV, E3 = 0.215 eV, E4 = 0.860 eV

Note the trivial solution with a constant wavefunction has a zero energy eigenvalue. Eigenenergy
E2 and E3 are degenerate with orthogonal (sine and cosine) wavefunctions.

Applied quantum mechanics 5


Solution 4

The first six eigenenergies are: E1 = 0.259 eV, E2 = 0.453 eV, E3 = 0.612 eV, E4 = 0.752 eV, E 5 =
0.884 eV, E6 = 1.02 eV

Solution 5
For a potential step V0 , impedance matching occurs when the ratio of effective electron masses
is
m2 E–V
------ = ---------------0
m1 E
or
V0
E = -----------------------------
-
1 – ( m2 ⁄ m 1 )
If the potential barrier energy is V0 = 1.5 eV and the ratio of effective electron mass on either
side of the heterointerface is m1 / m2 = 3, then
1.5
E = ----------------------- = 2.25 eV
1 – (1 ⁄ 3)
and the transmission coefficient in the limit E → ∞ is found by noting that
k2 m
---- = ------1
k1 E→ ∞
m2
so that

m1 4
Trans E→ ∞ = ------ ---------------------------2
m 2
1 + ------1
m
 m 2
For m 1 / m2 = 3 this gives 0.928 transmission and 0.072 reflection. One does not expect a particle
with infinite energy to be reflected by a finite potential step!

6
Applied Quantum Mechanics

Chapter 4 Problems and Solutions

LAST NAME FIRST NAME

Useful constants MKS (SI)

c = 2.99792458 × 10 m s
8 –1
Speed of light in free space
h = 6.58211889 ( 26 ) × 10
– 16
Planck’s constant eV s
h = 1.054571596 ( 82 ) × 10
–34
J s
e = 1.602176462 ( 63 ) × 10
– 19
Electron charge C
m 0 = 9.10938188 ( 72 ) × 10
–31
Electron mass kg
m n = 1.67492716 ( 13 ) × 10
–27
Neutron mass kg
Proton mass m p = 1.67262158 ( 13 ) × 10 –27 k g
Boltzmann constant k B = 1.3806503 ( 24 ) × 10 –23 J K –1
k B = 8.617342 ( 15 ) × 10 – 5 eV K –1
Permittivity of free space ε 0 = 8.8541878 × 10 – 12 F m –1
Permeability of free space µ 0 = 4π × 10 –7 H m –1
Speed of light in free space c = 1 ⁄ ε0µ0
Avagadro’s number N A = 6.02214199 ( 79 ) × 10 23 m o l –1
a B = 0.52917721 ( 19 ) ×10
–10
Bohr radius m
4πε 0 h 2
a B = ----------------
-
m 0e2

Inverse fine-structure constant α –1 = 137.0359976 ( 50 )


4 πε 0 hc
α – 1 = -----------------
-
e2

Applied quantum mechanics 1


PROBLEM 1
Write a computer program in Matlab that uses the propagation matrix method to find the trans-
mission resonances of a particle of mass m = m 0 (where m 0 is the bare electron mass).
(a) Use your computer program to find transmission as a function of energy for a particle mass
m0 through 12 identical one-dimensional potential barriers each of energy 1 eV, width 0.1 nm,
sequentially placed every 0.5 nm (so that the potential well between each barrier has width 0.4 nm).
What are the allowed (band) and disallowed (band gap) ranges of energy for particle transmission
through the structure? How do you expect the velocity of the transmitted particle to vary as a func-
tion of energy?
(b) How do these bands compare with the situation in which there are only three barriers, each
with 1 eV barrier energy, 0.1 nm barrier width, and 0.4 nm well width?
Your solution should include plots of transmission as a function of energy and a listing of the
computer program you used.

PROBLEM 2
Vertical-cavity surface-emitting lasers (VCSELs) operating at wavelength λ = 1300 nm are
needed for local area fiber optic applications.
(a) Use the propagation matrix to design a high-reflectivity Bragg mirror for electromagnetic
radiation with center wavelength λ0 = 1300 nm incident normal to the surface of an AlAs / GaAs
periodic dielectric layer stack consisting of 25 identical layer-pairs. Each individual dielectric layer
has a thickness λ / 4n, with n being the refractive index of the dielectric. Use n A l A s = 3.0 for the
refractive index of AlAs and n GaAs = 3.5 for GaAs. Calculate and plot optical reflectivity in the
wavelength range 1200 nm > λ > 1400 nm .
(b) Extend the design of your Bragg reflector to a two-mirror structure similar to that used in
the design of a VCSEL. This may be achieved by increasing the number of pairs to 50 and making
the thickness of the central GaAs layer one wavelength long. Recalculate and plot the reflectivity
over the same wavelength range as in (a). Using high wavelength resolution to find the bandwidth
of this optical pass band filter near λ = 1300 nm .

VCSEL structure center


wavelength λ0

nAlAs
nGaAs

25 identical
dielectric
Bragg mirror center layer pairs
wavelength λ 0

Center region
thickness
λ0 / n GaAs

25 identical 25 identical
dielectric dielectric
layer pairs layer pairs

n GaAs n GaAs
n AlAs n AlAs
z Incident electromagnetic field, Incident electromagnetic
wavelength λ field, wavelength λ

Your results should include a printout of the computer program you used and a computer-gener-
ated plot of particle transmission as a function of incident wavelength.

2
PROBLEM 3

1.0
3 nm 2.7 nm 2.25 nm
0.8

Potential energy, V(x) (eV)


0.6

0.4

0.2 0.5 eV

0.0 1.13 eV

-0.2
0.6 eV
-0.4

-0.6
0 5 10
Distance, x (nm)

Write a computer program in Matlab that uses the propagation matrix method to find the trans-
mission resonances of a particle of mass m = 0.07 × m 0 (where m 0 is the bare electron mass) for
the following one-dimensional potentials.
(a) A uniform electric field falls across the double barrier and single well structure as shown in
the Fig. The right-hand edge of the 2.25-nm-thick barrier of energy 1.13 eV is at a potential -0.6 eV
below the left-hand edge of the 3-nm-thick barrier of energy 0.5 eV. The well width is 2.7 nm.
Comment on the changes in transmission you observe.
(b) Rewrite your program to calculate transmission of a particle as a function of potential drop
caused by the application of an electric field across the structure. Calculate the specific case of ini-
tial particle energy E = 0.025 eV with the particle incident on the structure from the left-hand side.

PROBLEM 4
Write a computer program to solve the Schrödinger wave equation for the first 17 eigenvalues
of an electron with effective mass m e = 0.07 × m0 confined to the periodic potential sketched in
*

the following Fig. with periodic boundary conditions. Each of the eight quantum wells is of width
6.25 nm. Each quantum well is separated by a potential barrier of thickness 3.75 nm. The barrier
potential energy is 0.9 eV. How many energy bandgaps are present in the first 17 eigenvales and
what are their values? Plot the highest energy eigenfunction of the first band and the lowest energy
eigenfunction of the second band.

3.75 nm 6.25 nm
Potential energy, eV(x) (eV)

1.0

0.8

0.6

0.4

0.2

0.0
0 20 40 60 80 100

Distance, x (nm)

Applied quantum mechanics 3


PROBLEM 5
Use the results of Problem 4 with periodic boundary conditions to approximate a periodic one-
dimensional delta function potential with period 10 nm by considering 8 potential barriers with
energy 20 eV and width 0.25 nm. Plot the lowest and highest energy eigenfunction of the first
band. Explain the difference in the wave functions you obtain.

4
SOLUTIONS

Solution 1
Essentially the same as exercise 3 in chapter 4 (Chap4Exercise3 Matlab file) of the book.

Solution 2
Essentially the same as exercise 4 in chapter 4 (Chap4Exercise4 Matlab file) of the book.

Solution 3
Essentially the same as exercise 7 in chapter 4 (Chap4Exercise7 Matlab file) of the book.

Solution 4
This requires putting in periodic boundary conditions, but is otherwise the same as exercise 9 in
chapter 4 of the book. There are eight quantum wells and so there are eight eigenfunctions associ-
ated with each band. The first seventeen eigenfunctions include two complete bands each contain-
ing eight states. The eigenvalues and band gaps are:

E1 = 0.086418 eV
E2 = 0.086616 eV
E3 = 0.086616 eV
E4 = 0.087097 eV
E5 = 0.087097 eV
E6 = 0.087584 eV
E7 = 0.087584 eV
E8 = 0.087787 eV

Band gap = E9 - E8 = 0.247613 eV

E9 = 0.3354 eV
E10 = 0.33669 eV
E11 = 0.33669 eV
E12 = 0.33987 eV
E13 = 0.33987 eV
E14 = 0.34314 eV
E15 = 0.34314 eV
E16 = 0.34453 eV

Band gap = E17 - E16 = 0.35326 eV

E17 = 0.69779 eV

Applied quantum mechanics 5


0.9 0.08 ψ8 ψ9
Potential energy, V(x) (eV)

0.8 0.06

Wavefunction, ψ
0.7 0.04
0.6 0.02
0.5 0.00
0.4 -0.02
0.3 -0.04
0.2 -0.06
0.1
-0.08
0.0
0 10 20 30 40 50 60 70 80 0 10 20 30 40 50 60 70 80
Distance, x (nm) Distance, x (nm)

Solution 5
Use code from Problem 4. The difference in form of the wavefunction for lowest eigenenergy
value in the band (E 1 = 0.049316 eV) and highest eigenenergy value in the band (E8 = 0.053691
eV) is due to value of Bloch wave vector k in the wavefunction ψk (x) = uk (x) exp(ik.x) that
describes a particle in a potential of period L. For E 1 the value of k = 0 and for E8 the value of k =
π/L.

0.08
20 ψ1
0.06
Potential energy, V(x) (eV)

16 0.04
Wavefunction, ψ

0.02
12
0.00
8 -0.02
-0.04
4
-0.06 ψ8

0 -0.08
0 10 20 30 40 50 60 70 80 0 10 20 30 40 50 60 70 80
Distance, x (nm) Distance, x (nm)

6
Applied Quantum Mechanics

Chapter 5 Problems and Solutions

LAST NAME FIRST NAME

Useful constants MKS (SI)

c = 2.99792458 × 10 m s
8 –1
Speed of light in free space
Planck’s constant h = 6.58211889 ( 26 ) × 10 – 16 eV s
h = 1.054571596 ( 82 ) × 10
–34
J s
e = 1.602176462 ( 63 ) × 10
– 19
Electron charge C
m 0 = 9.10938188 ( 72 ) × 10
–31
Electron mass kg
m n = 1.67492716 ( 13 ) × 10
–27
Neutron mass kg
m p = 1.67262158 ( 13 ) × 10
–27
Proton mass kg
k B = 1.3806503 ( 24 ) × 10
–23 –1
Boltzmann constant JK
k B = 8.617342 ( 15 ) × 10
–5 –1
eV K
ε 0 = 8.8541878 × 10
– 12 –1
Permittivity of free space Fm
µ 0 = 4π × 10
–7 –1
Permeability of free space H m
Speed of light in free space c = 1 ⁄ ε0µ0
Avagadro’s number N A = 6.02214199 ( 79 ) × 10 23 m o l –1
Bohr radius a B = 0.52917721 ( 19 ) ×10–10 m
4πε 0 h 2
a B = ----------------
-
m 0e2

Inverse fine-structure constant α –1 = 137.0359976 ( 50 )


4 πε 0 hc
α – 1 = -----------------
-
e2

Applied quantum mechanics 1


PROBLEM 1
An electron in an infinite, one-dimensional, rectangular potential well of width L is in
the simple superposition state consisting of the ground and third excited state so that
1
ψ ( x, t ) = ------- ( ψ 1 ( x, t ) + ψ 4 ( x, t ) )
2
Find expressions for:
(a) The probability density, ψ ( x , t ) .
2

(b) The average particle position, 〈 x ( t ) 〉 .


(c) The momentum probability density, ψ ( p x , t )
2

(d) The average momentum, 〈 px ( t ) 〉 .


(e) The current flux, J ( x, t ) .

PROBLEM 2
(a) Show that the density of states for a free-particle of mass m in two-dimensions is
m
D2 ( E ) = ------------2
2πh
(b) At low temperature, electrons in two electrodes occupy states up to the Fermi
energy, EF. The two electrodes are connected by a two dimensional conductance
region. Derive an expression for the conductance of electrons flowing between the two
electrodes as a function of applied voltage V, assuming the transmission coefficient
through the two-dimensional region is unity. Consider the two limiting cases eV >> EF
and eV << EF

PROBLEM 3
Derive expressions for the two-dimensional D 2 ( ω ) and one-dimensional D1 ( ω )
opt opt

density of photon states in a homogeneous dielectric medium characterized by refrac-


tive index, n r.

PROBLEM 4
(a) In a particular system the dispersion relation for electrons in one-dimension is
Ek = hω k = 2t cos ( k x L ) , where t and L are constants and the wave vector in the x-
direction is 0 < k x < π ⁄ L . This dispersion relation can be derived using a nearest
neighbor tight binding model where t is the overlap integral between atomic orbitals.
Choosing one hundred equally spaced discrete values of k x, write a computer program
Γ ⁄π
and plot the electron density of states N ( E ) = ∑ ---------------------------------------------
using Γ = t ⁄ 10
( E – Ek ) + ( Γ ⁄ 2)
2 2
k

and t = – 1 .
(b) If one includes next nearest neighbor interactions, the dispersion relation in (a) can,
to within a scaling factor, be written E k = 2 t cos ( k x L ) + 2t′ cos ( 2k x L ) . Write a com-
puter program to plot the dispersion relation. Then calculate and plot the electron den-
sity of states using Γ = t ⁄ 10 , t = – 1 , t′ = – 0.2 and compare with the result you
obtained in (a) including a comparison with the effective electron mass at the band
edges.

2
(c) Write a computer program to plot the electron density of states for a square lattice
and cubic lattice both with lattice constant L and for which the dispersion relation is
E k = 2t ( cos ( k x L ) + cos ( k y L ) ) and E k = 2t ( cos ( k x L ) + cos ( ( k y L ) + cos ( k z L ) ) )
respectively. Use Γ = t ⁄ 10 and t = – 1 .
(c) The dispersion relation for a two dimensional hexagonal lattice is
E k = 2t ( 2 cos ( 0.5 × k x L ) cos ( 0.866 × k y L ) + cos ( k x L ) )
+ 2t′ ( 2 cos ( 0.866 × k y L ) cos ( 1.5 × k x L ) + cos ( 1.73 × k y L ) )
where t′ is the overlap integral with the next nearest neighbor. Calculate the electron
density of states using Γ = t ⁄ 10 , t = – 1 , and t′ = 0.3 . Compare with the result
you obtain when t′ = 0 .

PROBLEM 5
A hydrogen atom is in its ground state with electron wave function
1⁄2
⋅ e B ⋅  ------ 
2 –r ⁄ a 1
φ 100 = -----------
aB
3⁄2  4 π
In this expression aB is the Bohr radius and r is a radial coordinate.
Use spherical coordinates to find the expectation value of position r and momentum p r
for the electron in this state. You should use the fact that in radial coordinates the Her-
1∂
mitian momentum operator is p̂ r = – ih -- r .
r ∂r

PROBLEM 6
Using the fact that the Hamiltonian H appearing in the Schrödinger equation

----H |ψ 〉 = |∂ψ〉
–i
h ∂t
is Hermitian (i.e., 〈ψ |Hψ〉 = 〈Hψ |ψ 〉 ), show that the time dependence of the average
value of the operator  is
d i ∂
--- 〈 Â〉 = -- 〈 [H , Â]〉 + 〈 Â〉
dt h ∂t

PROBLEM 7
Show that:
(a) The position operator x̂ acting on wavefunction ψ ( x ) is Hermitian (i.e., x̂ † = x̂ ).
d
(b) The operator acting on the wavefunction ψ ( x ) is anti-Hermitian (i.e.,
dx

 d  = – d ).
dx dx
d
(c) The momentum operator – i h acting on the wavefunction ψ ( x ) is Hermitian.
dx

PROBLEM 8
Classical electromagnetic theory uses real magnetic and electric fields coupled via
Maxwell’s equations. The magnetic and electric fields each have physical meaning.
Both fields are needed to describe both the instantaneous state and time evolution of the

Applied quantum mechanics 3


system. Quantum mechanics uses one complex wave function to describe both the
instantaneous state and time evolution of the system. It is also possible to describe
quantum mechanics using two coupled real wave functions corresponding to the real
and imaginary parts of the complex wave function. However, such an approach is more
complicated. In addition, the real and imaginary parts of the wave function have no
special physical meaning.
(a) If classical electromagnetism were described in terms of a single complex field
G = ε 0 ⁄ 2 ⋅ E + i µ 0 ⁄ 2 ⋅ H show that Maxwell’s equations in free space and in the
absence of free charges may written as the complex equations
∇⋅G = 0
and
∂G 1
i = -------------- ∇ × G
∂t ε0µ 0
(b) Show that the energy flux density in the electromagnetic field given by the Poynting
vector is
–i
S = E × H = -------------- ( G × G )
*

ε0µ 0
(c) If the field G is purely real, what is the value of S?
2
(d) Show that the electromagnetic energy density is U = G .

4
SOLUTIONS

Solution 1
(a) Let the potential in the region – L ⁄ 2 < x < L ⁄ 2 be zero and infinity elsewhere. The
– iω nt
eigenfunctions are ψ n ( x, t ) = 2 ⁄ L sin ( k n ( x + L ⁄ 2 ) )e , where k n = nπ ⁄ L ,
ω n = hk ⁄ 2m , and n = 1 , 2, 3, … . Hence,
2
n
–iω 1 t –i ω1 t
ψ 1 (x , t ) = 2 ⁄ L sin ( ( π ⁄ L ) ( x + L ⁄ 2 ) ) e = 2 ⁄ L cos ( ( πx ⁄ L ) e )
–i ω4 t –i ω4 t
ψ 4 ( x , t ) = 2 ⁄ L sin ( ( 4 π ⁄ L ) ( x + L ⁄ 2 ) ) e = 2 ⁄ L sin ( 4πx ⁄ L ) e
and the probability distribution of the particle in the superposition state
ψ ( x, t ) = 1 ⁄ 2 ( ψ 1 ( x, t ) + ψ 4 ( x , t ) )
is
1
ψ ( x, t ) 2
= -- ( ψ 1 ( x ) 2 + ψ 4 ( x ) 2 + 2 ψ 1 ( x ) ψ 4 ( x ) cos ( ( ω 4 – ω 1 )t ) )
2
4 πx πx h 15π  
2
= --- cos 2 ------  + sin2  ---------  + 2 cos  ------  sin  ---------  cos  ------- -----------t
1 xπ 4πx
ψ ( x, t )
2

L L  L  L  L   2m L 2  

h 15 π
2
where we note that ω 4 – ω 1 = ------- ----------
-.
2m L 2
(b) Expectation value of position is
〈 x ( t )〉 = ∫ ψ ( x, t)
2
x dx
L⁄2
1
〈 x ( t )〉 = -- ∫ ( ψ 1 ( x ) x + ψ 4 ( x ) x + 2x ψ 1 ( x ) ψ 4 ( x ) cos ( ( ω 4 – ω 1 ) t ) ) dx
2 2
2 –L ⁄ 2
By symmetry, the first two terms in the integrand are zero, leaving
L⁄2
πx 4 πx
--- cos  ------  sin  ---------  x d x
2
〈 x ( t ) 〉 = cos ( ( ω 4 – ω 1 )t ) ∫ L L L
–L ⁄ 2

making use of 2 sin ( x ) cos ( y ) = sin ( x + y ) + sin ( x – y ) gives


L⁄2
πx 
 sin  5--------
- + sin  ---------  x dx
1 3π x
〈 x ( t ) 〉 = cos ( ( ω 4 – ω 1 ) t ) -- ∫
L   L   L 
–L ⁄ 2
which, after integrating by parts, results in
2 2
〈 x ( t ) 〉 = cos ( ( ω 4 – ω 1 )t ) ---  -----------2 – --------2  x dx
1 2L 2L
L  25π 9 π 
32 L
〈 x ( t )〉 = – --------------2 cos ( ( ω 4 – ω 1 ) t )
225π
(c) The momentum probability density ψ ( p x, t ) 2 is found using the Fourier transform
L⁄2
1 – ip x 1 – ip x –i p x
ψ ( p x, t ) = -------------- ∫ ψ ( x , t )e x d x = -------------- ∫ ( ψ 1 ( x, t )e x + ψ 4 ( x, t ) e x ) dx
2πh 4π h – L ⁄ 2
L⁄2 L⁄2
ψ ( p x, t ) = ------------- e 1
2 – iω t
 π x-  e –i px x d x + e –i ω4 t sin  ---------  e x dx 
4πx –i p x
4 πhL  ∫ cos  -----
L  ∫  L 
–L ⁄ 2 –L ⁄ 2

Applied quantum mechanics 5


L⁄2 L⁄ 2
π x ip x
= -------------  e 1 ∫ cos  ------  e x dx + e 4 ∫ sin  ---------  e x d x ×
1 iω t iω t 4πx ip x
ψ ( p x, t ) 2

2πh L   L   L  
–L ⁄ 2 –L ⁄ 2
L⁄2 L⁄2
 – iω1 t  πx- e –ip xx dx + e – iω4 t 4 πx –i px x 
sin  --------- e
e ∫ cos  -----
L  ∫ L
d x
–L ⁄ 2 –L ⁄ 2

L⁄2 2 L⁄2 2
π x –i px x
cos  ------  e sin  --------- e
1 4πx – ip xx
ψ ( p x, t ) 2
= ------------- ∫ dx + ∫ dx +
2πh L  L  L 
–L ⁄ 2 –L ⁄ 2
L⁄2
πx
2Re  e cos  ------ e sin  --------- e d x
i ( ω4 – ω1 )t –ip x x 4πx i px x
∫ L L
–L ⁄ 2

We make use of the standard indefinite integrals (I. S. Gradshteyn and I. M. Ryzhik,
Table of integrals, series, and products, Academic Press, San Diego, 1980, p. 196
(ISBN 0 12 294760 6))
e ( a cos ( bx ) + b sin ( bx ) )
ax
cos ( bx ) dx = --------------------------------------------------------------
ax
∫e 2 2
-
a +b
e ( a sin ( b x ) – b cos ( b x ) -)
ax

∫e sin ( bx ) dx = --------------------------------------------------------------
ax
2 2
a +b
ip L ⁄ 2 – ip L ⁄ 2 2 ip L ⁄ 2 –i p L ⁄ 2 2
1 Lπ ( e x + e x ) π( e x – e x ) +
ψ ( p x, t ) + 4L
2
= ------------- ------------------------------------------------ ---------------------------------------------------
2πh L π – px L
2 2 2
16 π – p x L
2 2 2

ip L ⁄ 2 –i p L ⁄ 2 –i p L ⁄ 2 ip L ⁄ 2
 i ( ω4 – ω1 )t Lπ ( e x + e x ) 4 Lπ ( e x – e x ) 
2Re  e ------------------------------------------------ -
--------------------------------------------------
 π 2 – p 2x L 2 16 π 2 – p 2x L 2 

= A cos 2  --------  + Bsin 2  --------


p xL p xL 
ψ (p x, t ) + C sin ( p x L ) sin ( ( ω 4 – ω 1 ) t )
2

2 2 
2Lπ 32L π –8L π
A = -------------------------------
- , B = ----------------------------------------2 , C = ----------------------------------------------------------------- .
h( π – L px )
2 2 2
h ( ( 4π ) – L p x ) h ( π – L p x ) ( ( 4π ) – L p x )
2 2 2 2 2 2 2 2 2 2

(d) The expectation value of momentum is


L⁄2
∂ –i h ( ψ * + ψ * ) ∂ ( ψ + ψ )  d x
〈 px ( t ) 〉 = –ih ∫ ψ ( x , t ) ψ ( x, t ) dx = -------- ∫
*

∂x 2  1 4
∂x 1
4 
–L ⁄ 2

L⁄2 L⁄2
∂ψ 4 ∂ψ 1
ψ4 ψ1
* *
The terms ∫ ∂x
d x = 0 and ∫ ∂x
dx = 0 by symmetry, leaving
–L ⁄ 2 –L ⁄ 2

L⁄2
– i h 2 4 π – i( ω4 – ω1 )t πx
cos  ------ cos  --------- dx +
4πx
〈 p x ( t )〉 = -------- -- ------ e ∫
2 LL  L  L 
–L ⁄ 2
L⁄2
ih 2 π i ( ω4 – ω1 )t  4 πx- sin  πx
----- -- -- e ∫ sin  -------- ------ d x
2 LL –L ⁄ 2
L   L

– i h 8π – i( ω 4 – ω1 )t 2L ih 2π i( ω4 – ω1 )t 8L
〈 p x ( t )〉 = -------- ------2 e --------- + ----- ------2 e ---------
2 L 15 π 2 L 15π
– i h8 – i( ω – ω )t i (ω – ω ) t
〈 p x ( t )〉 = ----------- ( e 4 1 + e 4 1 )
15L
and

6
16h
〈 p x ( t )〉 = --------- sin ( ( ω 4 – ω 1 )t )
15 L
(e) The current operator is
– ie h * ∂ψ ∂ψ * 
J ( x , t ) = -----------  ψ –ψ
2m ∂x ∂x 
– ie h * ∂ ∂
J ( x , t ) = -----------  ( ψ 1 + ψ 4 ) ( ψ 1 + ψ 4 ) – ( ψ 1 + ψ 4 ) ( ψ 1 + ψ 4 ) 
* * *

4m ∂x ∂x
πx ∂
J ( x, t ) = -----------  cos  ------  sin  ---------  ( e
– i eh 4π x i( ω1 – ω4 )t – i( ω1 – ω4 )t
–e )+
2L m L ∂x L
4πx ∂ πx –i ( ω1 – ω4 )t
sin  --------- cos  ------ ( e )
i( ω 1 – ω4 )t
–e
L ∂x L
–e h πx ∂ 4π x ∂ πx
J ( x, t ) = ---------  cos  ------  sin  ---------  – sin  ---------  cos  ------  sin ( ( ω 4 – ω 1 ) t )
4π x
Lm   L ∂x   L   L ∂x   L 
–πe h
4 cos  π ---  cos 4 π ---  – sin  4π --  sin  π ---   sin ( ( ω 4 – ω 1 )t )
x x x x
J ( x, t ) = ------------
2 
mL L  L   L   L 

Solution 2
(a) Density of states of particle mass m in two dimensions is
m
D 2 ( E ) = ------------2 per spin.
2π h
(b) Current is assumed proportional to applied voltage, electron velocity v ( E ) , trans-
mission coefficient T ( E ) , and density of states.
EF + e V E F + eV

v ( E )T ( E )D 2 ( E ) d E = e  --h-- ----------
2mE   -----------
m -
I = e ∫ ∫  m h 2 -  T ( E )  2 πh 2  dE
EF EF

If we assume T ( E ) = T ( E F ) = 1 , then
EF + e V
eV 3 ⁄ 2
= ------------2 E F  1 + ------  – 1 
2m 2m 2 3 ⁄ 2 EF + eV 2m 3 ⁄ 2
I = e ------------2 ∫ E d E = ------------2 ⋅ - [ E ] EF
2πh EF 2π h 3 3πh EF

When E F « eV , conductance per electron spin

G = --- =  ------------2 V
I 2m
V  3 πh 
For the other situation when EF » e V we use the binomial expansion
eV 3 ⁄ 2
( 1 + x ) = 1 + nx + … so that 1 + ------ 
n 3 eV
= 1 + -- ------ . Conductance per electron
EF 2 EF
spin
I 2m
G = --- = ------------2 E F
V 2πh

Solution 3
The two dimensional and one dimensional photon density of states in an isotropic
medium of refractive index n r is

Applied quantum mechanics 7


ωn
2
D2 ( ω ) = --------2-r
πc
nr
D1 ( ω ) = ------
πc

Solution 4
(a) We are given the dispersion relation for electrons in one-dimension as
Ek = 2t cos ( k x L ) , where t = – 1 and 0 < k x < π ⁄ L . We are asked to plot the density
Γ ⁄π
of states N ( E ) = ∑ ---------------------------------------------
using Γ = t ⁄ 10 and a sum over 100 states.
k ( E – E k ) + (Γ ⁄ 2 )
2 2

Notice that the cosine dispersion in one-dimension has a van-Hove singularity the band
edge energies of ±2t.

% HW5Exercise4a
clear all;
clf;

natoms=100; %number of lattice points (atoms) in each direction


npoints=100; %number of points in plot
emax=6.5; %max energy in units of t
emin=-6.5; %min energy in units of t
if emax<=emin;error('Error: require emax > emin');end;
estep=(emax-emin)/(npoints-1);

dos=zeros(npoints); %set dos vector to zero


w=emin:estep:emax; %energy
t=-1; %set energy scale - for s-orbital, + for p-orbital
gamma = 0.1; %Energy broadening in units of t
gamma=gamma*abs(t); %use units of t
gamma2=(gamma/2)^2;
const=gamma/pi;
kx0=pi/(natoms); %kx constant
kx=0:kx0:kx0*natoms;

for ix=1:natoms+1
% Nearest neigbor dispersion for the linear lattice
Ek(ix)=2.0*t*cos(kx(ix));
for j=1:npoints
dos(j)=dos(j)+const/((w(j)-Ek(ix))^2.+gamma2);
end
end

figure(1);
plot(w,dos);
ttl=['Nearest neighbor tight binding dispersion, \gamma =',num2str(gamma),'t, natoms=',num2str(natoms)];
title(ttl);
xlabel('Energy, E (t)'),ylabel('Density of states, N(E)');

8
figure(2);
plot(Ek);
title(ttl);
xlabel('Wave vector, k_x ( \Deltax \pi/L)'),ylabel('Energy, E_k(t)');

(b) The one-dimensional density of states increases at high energy band edge and there
is a corresponding increase in effective electron mass in this energy range. When we
include next nearest neighbors the density of states is no longer symmetric in energy.
The next nearest energy interaction pushes eigenvalues to higher energy.
% HW5Exercise4b
clear all;
clf;

natoms=100; %number of lattice points (atoms) in each direction


npoints=100; %number of points in plot
emax=6.5; %max energy in units of t
emin=-6.5; %min energy in units of t
if emax<=emin;error('Error: require emax > emin');end;
estep=(emax-emin)/(npoints-1);

dos=zeros(npoints); %set dos vector to zero


w=emin:estep:emax; %energy
t=-1; %set energy scale - for s-orbital, + for p-orbital
tprime=-0.2 %tprime
gamma = 0.1; %Energy broadening in units of t
gamma=gamma*abs(t); %use units of t
gamma2=(gamma/2)^2;
const=gamma/pi;
kx0=pi/natoms; %kx constant
kx=0:kx0:kx0*natoms;

for ix=1:natoms+1
% Nearest neigbor dispersion for the linear lattice
Ekprime(ix)=(2.0*t*cos(kx(ix)))+(2.0*tprime*cos(2*kx(ix)));
Ek(ix)=2.0*t*cos(kx(ix));
for j=1:npoints
dos(j)=dos(j)+const/((w(j)-Ekprime(ix))^2.+gamma2);
end
end

figure(1);
plot(w,dos);
ttl=['Nearest neighbor tight binding dispersion, \gamma =',num2str(gamma),'t, natoms=',num2str(natoms)];
title(ttl);
xlabel('Energy, E (t)'),ylabel('Density of states, N(E)');
figure(2);
plot(Ek,'b');
hold;
plot(Ekprime,'r');
title(ttl);
xlabel('Wave vector, k_x ( \Deltax \pi/L)'),ylabel('Energy, E_k(t)');
hold off;

Applied quantum mechanics 9


(c) For the square lattice with nearest neighbor interactions only, we have

and for the cubic lattice

Notice that the band edge energy for simple cubic lattice of dimension d occurs at ±2td.

(d) For the hexagonal lattice with second nearest neighbor interactions

which is different compared to the result for the hexagonal lattice with only nearest
neighbor interactions shown in the following figure.

10
Solution 5
∞ ∞ ∞
4 – 2r ⁄ aB 3
〈 r〉 = ∫ φ 100 rφ 100 d r =
3
∫ φ 100 φ 100 4 πr dr = -----
* * 3
3∫
e r dr
0 0 aB 0
Let x = 2 r ⁄ a B , so that

a 3 –x a B 3! 3
〈 r〉 = -----B ∫ x e dx = ---------- = - aB
40 4 2

n –µ x
dx = n! µ for Re ( µ > 0 ) .
–n –1
where we made use of the standard integral ∫ x e
0

∞ ∞
1d 1d
〈 p r〉 = – ih ∫ φ *100 -- ( r φ 100 ) 4πr d r = – ih ∫ φ 100 -- ( rφ 100 ) 4πr dr = – ihA
2 2

0
r d r 0
r d r
where A is a real number. Because the measurable quantity 〈 p r〉 must be a real num-
ber, A = 0 and hence 〈 p r〉 = 0

Solution 6
The Hamiltonian H in the Schrödinger equation is Hermitian and so it is its own Hermi-
tian adjoint, i.e., H = H† and 〈ψ |Hψ 〉 = 〈 Hψ|ψ 〉 . The time dependence of the expe-
ˆ is
cation value of the operator A
d ˆ
----- 〈 A ˆ |ψ 〉 + 〈ψ | ∂ A
〉 = 〈 ∂ψ| A ˆ |ψ〉 + 〈ψ |Â |∂ψ 〉
dt ∂t ∂t ∂t
d ˆ
--- 〈 A
i i ∂ ˆ
〉 = -- 〈Hψ |Â |ψ 〉 – -- 〈ψ |ÂH |ψ 〉 + 〈 ψ| A |ψ 〉
dt h h ∂t
d i ˆ |ψ 〉 – -i- 〈ψ |ÂH |ψ 〉 + 〈 ψ| ∂ Â |ψ 〉
--- 〈 Â〉 = -- 〈ψ |HA
dt h h ∂t
d i ∂
--- 〈 Â〉 = -- 〈 [H , Â]〉 + 〈 Â〉
dt h ∂t

Applied quantum mechanics 11


Solution 7
(a) The position operator x̂ is Hermitian if it is its own Hermitian adjoint, i.e., x̂† = x̂
or 〈ψ |x | φ〉 = 〈φ |x |ψ〉 . For the operator x̂ this is easy to show
*

∞ ∞ ∞

〈ψ| xψ 〉 = ∫ψ ( x ) ( xψ ( x ) ) d x = ∫ ( xψ ( x )) ψ( x )d x = ∫ ( xψ ( x ) ) ψ ( x ) dx = 〈x ψ|ψ 〉
* * *

–∞ –∞ –∞

d
(b) To show that the operator is anti-Hermitian we integrate by parts and make use
dx
of the fact that the wavefunction ψ ( x ) → 0 as x → ±∞
∞ ∞

〈 ψ|
d
ψ〉 =  d  x=∞ d (x )ψ (x )d x
∫ ψ ( x )  ih d x ψ ( x )  d x = ψ ( x )ψ ( x ) ∫ dx ψ
* * *

dx x = –∞ 
–∞ –∞

∞ *
ψ 〉 = – ∫  ψ ( x ) ψ ( x ) dx = – 〈 ψ| ψ〉
d d d
〈 ψ|
dx –∞
d x d x

and   = –
d d
.
d x dx
d
(c) Because the operator is anti-Hermitian, it follows that the momentum operator
dx

is Hermitian since  – i h  = ih –  = – i h .
d d d d
– ih
dx  dx  d x dx

Solution 8
(a) ∇ ⋅ G = ∇ ⋅ ( E ε 0 ⁄ 2 + i H µ 0 ⁄ 2 ) = 0

∂G ε ∂E µ 0 ∂H 1 1 1
i = i ----0 – ----- = -------------- ∇× G = ------------- ∇×E + i ------------ ∇×H
∂t 2 ∂t 2 ∂t ε0µ0 2µ 0 2ε 0
Evaluating real and imaginary parts gives
∂H
∇× E = – µ 0
∂t
and
∂E
∇× H = ε 0
∂t
(b) We start with
ε µ ε µ
-------------- G × G = --------------  ----0 E – i -----0 H  ×  ----0 E + i -----0 H 
–i * –i
ε0µ0 ε0µ0  2 2   2 2 

–i ε µ ε0µ0 ε0µ0
-------------- G * × G = --------------  ----0 E × E + -----0 H × H + i ---------- H × E
–i
E × H – i ----------
ε0µ 0 ε0µ0 2  2 4 4 

ε0µ0 ε0µ0
-------------- G × G = --------------  i -------------- E × H = E × H = S
–i –i
E × H + i --------------
*

ε0µ 0 ε0µ0  2 2 

(c) If G is purely real then G = G and G × G = 0 , so S = 0 .


* *

12
ε µ ε µ
(d) G ⋅ G =  ----0 E – i -----0 H  ⋅  ----0 E + i -----0 H 
*
2 2 2 2
ε0 2 µ0 2 ε0µ 0 ε0µ0 1
G ⋅ G = ---- E + ----- H – i ---------- E ⋅ H + i ---------- H ⋅ E = -- ( E ⋅ D + B ⋅ H ) = U
*
2 2 2 2 2
Another way of looking at this is to recall that
S
U = ------
c
Now, because we consider an electromagnetic wave in free-space, the speed of light
c = 1⁄ ε 0 µ 0 and θ = 90° , and we have
1 G ×G
*
S c *
U = ------ = -------------- -------------------- = -- G G sin θ = G
2

c ε0µ0 c c

Applied quantum mechanics 13


14
Applied Quantum Mechanics

Chapter 6 Problems and Solutions

LAST NAME FIRST NAME

Useful constants MKS (SI)

c = 2.99792458 × 10 m s
8 –1
Speed of light in free space
Planck’s constant h = 6.58211889 ( 26 ) × 10 – 16 eV s
h = 1.054571596 ( 82 ) × 10
–34
J s
e = 1.602176462 ( 63 ) × 10
– 19
Electron charge C
m 0 = 9.10938188 ( 72 ) × 10
–31
Electron mass kg
m n = 1.67492716 ( 13 ) × 10
–27
Neutron mass kg
m p = 1.67262158 ( 13 ) × 10
–27
Proton mass kg
k B = 1.3806503 ( 24 ) × 10
–23 –1
Boltzmann constant JK
k B = 8.617342 ( 15 ) × 10
–5 –1
eV K
ε 0 = 8.8541878 × 10
– 12 –1
Permittivity of free space Fm
µ 0 = 4π × 10
–7 –1
Permeability of free space H m
Speed of light in free space c = 1 ⁄ ε0µ0
Avagadro’s number N A = 6.02214199 ( 79 ) × 10 23 m o l –1
Bohr radius a B = 0.52917721 ( 19 ) ×10–10 m
4πε 0 h 2
a B = ----------------
-
m 0e2

Inverse fine-structure constant α –1 = 137.0359976 ( 50 )


4 πε 0 hc
α – 1 = -----------------
-
e2

Applied quantum mechanics 1


PROBLEM 1
(a) Write down the Hamiltonian for a particle of mass m in a one-dimensional har-
monic oscillator potential in terms of momentum p x and position x.
(b) If one defines new operators
1⁄ 2
b̂ =  --------  x̂ + -------
mω i p̂ x 
-
 2h   mω
1⁄ 2
b̂ =  --------  x̂ – -------x- 
† mω i p̂
 2h   mω 
show that the Hamiltonian can be expressed as
hω † †
H = ------- ( b̂b̂ + b̂ b̂ )
2
(c) Derive the commutation relation [ b̂, b̂ ] .

(d) Using your result from (c) to show that the Hamiltonian is

H = hω  b̂ b̂ + -- 
† 1
2

PROBLEM 2
(a) Find the expectation value of position and momentum for the first excited state
for a particle of mass m in a one-dimensional harmonic oscillator potential.
(b) Find the value of the product in uncertainty in position ∆x and momentum ∆px
for the first exited state of a particle of mass m in a one-dimensional harmonic oscillator
potential.

PROBLEM 3
ˆ is time-independent but the corresponding numerical value of the
Often an operator A
observable A has a spread in values ∆A about an average value 〈 A ( t ) 〉 and varies with
time because the system is described by a wavefunction ψ ( x , t ) which is not an eigen-
d
state. The change in 〈 A ( t ) 〉 in time interval ∆t is the slope〈 A ( t ) 〉 multiplied by ∆t.
dt
Hence, the exact time t at which the numerical value of the observable A passes through
a specific value will actually have a spread in values ∆t such that
d
∆t = ∆ A ⁄ 〈 A〉
dt
i
(a) Use the generalized uncertainty relation ∆A ∆B ≥ -- 〈 [ Â, B̂ ]〉 for time independent
2
h
operators  and B̂ to show that ∆E∆ t ≥ --- .
2
(b) Show that the spread in photon number ∆n and phase ∆φ for light of frequency ω
is
1
∆n∆φ ≥ --
2
and that for a Poisson distribution of such photons
1
∆φ ≥ ----------------
2 〈 n〉

2
PROBLEM 4
A particle of charge e, mass m, and momentum p oscillates in a one-dimensional har-
monic potential V ( x̂ ) = mω 0 x̂ ⁄ 2 and is subject to an oscillating electric field
2 2

E x cos ( ω t ) .
(a) Write down the Hamiltonian of the system.

(b) Find d 〈 x̂〉 .


dt
d d d
(c) Find 〈 p̂〉 and show that 〈 p̂〉 = – 〈 V ( x̂ ) 〉 . Under what conditions is the
dt dt dx
2
d d
quantum mechanical result m 2
〈 x̂〉 = – 〈 V ( x̂ )〉 the same Newton’s second law in
dt d x
2
dx
which force on a particle is F = m = – d V ( x) ?
d t2 dx

(d) Find d 〈 H〉 .
dt
(e) Use your results in (b) and (c) to find the time dependence of the expectation value
of position 〈 x̂ 〉 ( t ) . What happens to the maximum value of 〈 x̂〉 as a function of time
when ω 0 = ω and when ω is close in value to ω 0 ?

Applied quantum mechanics 3


SOLUTIONS

Solution 1
(a) The Hamiltonian for a particle of mass m in a one-dimensional harmonic oscillator
potential is
2
p̂ mω 2 2
H = -------x + ---------- x̂
2m 2
where the particle moves in the potential V ( x ) = κx
2
and oscillates at frequency

ω = κ ⁄ m , where κ is a force constant. The operator p̂ x = – i h ----- .
∂x
(b) The Hamiltonian can be factored
2
mω 2 p̂ x
H = ----------  ------------ + x̂ 
2

2 m 2ω2
so it makes sense to define new operators
1⁄ 2
b̂ =  --------  x̂ + -------
mω i p̂ x 
-
2h  mω
1⁄ 2
b̂ =  --------  x̂ – -------
† mω i p̂ x 
-
2h  mω 
which, can be written in terms of operators x̂ and p̂ x to give
h 1⁄2
x̂ =  ------------  ( b̂ + b̂ )

2mω
hmω
1⁄ 2
p̂ x = i  ------------

( b̂ – b̂ )
2
Substituting this into the Hamiltonian gives
–1 hmω h
2 2 2
H = -------x + ---------- x̂ = -------  ------------  ( b̂ – b̂ ) + ----------  ------------ ( b̂ + b̂ )
p̂ mω 2 † 2 mω † 2

2m 2 2m 2 2 2 mω
hω h ω
H =  ------- ( – b̂ b̂ – b̂ b̂ + b̂b̂ + b̂ b̂ ) +
† † † † † † † †
 ------- ( b̂b̂ + b̂ b̂ + b̂b̂ + b̂ b̂ )
4 4
hω † †
H = ------- ( b̂b̂ + b̂ b̂ )
2
(c) To derive the commutation relation [ b̂, b̂ ] = b̂b̂ – b̂ b̂ we can write out the differ-
† † †

ential form of the operators and have them act on a dummy wave function. This gives
h ∂ h ∂
( b̂ b̂ ) ψ =  --------  x + -------- -----  x – -------- -----  ψ
† mω
 2h   mω ∂ x  mω ∂x 

h h ∂ h ∂ h -------∂ -
2 2
( b̂ b̂ ) ψ =  --------  x + -------- + -------- x ----- – -------- x ----- – ------------
† mω
ψ
2

2h mω mω ∂ x mω ∂x m2 ω 2 ∂x 2
and
h ∂ h ∂
( b̂ b̂ ) ψ =  --------  x – -------- -----   x + -------- -----  ψ
† mω
 2h   mω ∂ x  mω ∂x 

4
h h ∂ h ∂ h ∂ 
2 2
( b̂ b̂ ) ψ =  --------  x – -------- – --------x ----- + -------- x ----- – ------------
† mω
-------- ψ
2

2h mω mω ∂x mω ∂x m 2 ω 2 ∂x 2
so that the commutation relation simplifies to just
2h ∂ 2h ∂
( b̂ b̂ – b̂ b̂ ) ψ =  --------   -------- – -------- x ----- + -------- x -----  ψ = ψ
† † mω 2h
2h mω mω ∂x mω ∂ x
or, in even more compact form,
† † †
[ b̂ , b̂ ] = b̂b̂ – b̂ b̂ = 1
If we do not want to use differential operators and a dummy wavefunction then we
could write

[ b̂ , b̂ ] = b̂b̂ – b̂ b̂ =  --------  x̂ + ----------x  x̂ – ----------x  –  --------   x̂ – ----------x   x̂ + ----------x


† † † mω ihp̂ ihp̂ mω ihp̂ ih p̂
 2h   mω   mω   2h   mω   mω 
ihp̂ x x̂ ihx̂p̂ x h p̂ x   mω  2 ihp̂ x x̂ ihx̂ p̂x h p̂ x 
2 2 2 2
[ b̂ , b̂ ] =  --------  x̂ 2 + -------------
† mω
- – -------- x̂ – ------------- + ------------- + -----------
– ------------- + ----------- -
 2h   mω mω m 2 ω 2  2h   mω mω m 2 ω 2
imω hp̂ x x̂ hx̂ p̂ x
[ b̂ , b̂ ] =  ----------   ----------- – ----------- = i ( p̂ x x̂ – x̂p̂ x ) = i [ p̂ x,x̂ ] = i ( – i h ) = 1

h mω mω
since [p̂ x ,x̂] = – i h .
(d) The Hamiltonian is
hω hω hω
H = ------- ( b̂b̂ + b̂ b̂ ) = ------- ( b̂b̂ + b̂ b̂ + b̂ b̂ – b̂ b̂ ) = ------- ( 1 + 2 b̂ b̂ ) = hω  b̂ b̂ +
† † † † † † † † 1
---
2 2 2  2
† † †
where we made use of the commutation relation [ b̂, b̂ ] = b̂b̂ – b̂ b̂ = 1

Solution 2
(a) The expectation value of position 〈 x〉 and momentum 〈 p 〉 for the first excited state
| n = 1〉 of a particle of mass m in a one-dimensional harmonic oscillator potential is
found using
h
1⁄2
x̂ =  ------------

( b̂ + b̂ )
2mω
h mω 1 ⁄ 2 †
p̂ x = i  ------------ ( b̂ – b̂ )
2
and
† 1⁄2
| b̂ n〉 = ( n + 1 ) |n + 1 〉
1⁄ 2
| b̂n〉 = n |n – 1〉
〈 m| n 〉 = δ mn
so that
h 1⁄2 h 1⁄ 2
〈 x〉 = 〈1| x |1 〉 =  ------------  〈 1|b̂ + b̂ |1〉 =  ------------ ( 〈 1|0 〉 +

2 〈1 |2〉 ) = 0
 2mω   2mω
and
hmω 1 ⁄ 2 h mω 1 ⁄ 2
〈 p̂ x〉 = 〈1| p̂ x |1 〉 = i  ------------ 〈1| ( b̂ – b̂ ) |1〉 = i  ------------  ( 2 〈1|2〉 – 〈1| 0〉 ) = 0

 2   2 

Applied quantum mechanics 5


(b) To find the value of the product in uncertainty in position ∆x and momentum ∆px for
the first exited state of a particle of mass m in a one-dimensional harmonic oscillator
potential we use
∆x = ( 〈 x 2〉 – 〈 x〉 2 ) 1 / 2
and
∆p x = ( 〈 p 2 x〉 – 〈 px 〉 2 ) 1 / 2
and since 〈 x〉 = 0 and 〈 p x 〉 = 0 we will be interested in finding the value of 〈 x 2〉
and 〈 p 2 x〉 . Starting with 〈 x 2〉 , we have
h h
〈 x̂ 2〉 =  ------------  〈1 | ( b̂ + b̂ ) | 1〉 =  ------------ 〈 1|b̂ b̂ + b̂ b̂ + b̂b̂ + b̂ b̂ |1 〉 = ------------
† 2 † † † † 3h
 2mω   2 mω 2 mω
h
and one can see that for the general state |n〉 one has 〈 x 〉 =  ------------  ( 1 + 2 n ) . Now
2
 2mω 
turning our attention to 〈 p 2 x〉 we have
h mω hmω
〈 p 2x 〉 =  ------------ 〈1 | ( b̂ – b̂ ) | 1〉 =  ------------ 〈 1| – b̂b̂ – b̂ b̂ + b̂b̂ + b̂ b̂ | 1〉 = ---------------
† 2 † † † † 3h mω
2  2  2
h mω
and one can see that for the general state |n 〉 one has 〈 p̂ x 〉 =  ------------  ( 1 + 2n ) .
2

2
For the particular case we are interested in | n = 1〉 and the uncertainty product is
3
∆x ∆p x = ( 〈 x 2〉 〈 p 2x 〉 ) 1 ⁄ 2 = - h
2
h
For the general state | n〉 the uncertainty product ∆x ∆p x = -- ( 1 + 2 n )
2

Solution 3
(a) We start with the reasonable assumption that the expectation value of an observable
associated with operator  evolves smoothly in time such that
d ∆A
〈 Â〉 = -------
dt ∆t
which may be written as
∆A
∆t = ------------------
d ˆ
〈 A〉
dt
ˆ is time-independent so that
In this problem the operator A
d i ∂ i
〈 Â〉 = -- 〈 [ H ,Â]〉 + 〈 Â〉 = -- 〈 [ H ,Â]〉
dt h ∂t h
∂ ˆ
since 〈 A〉 = 0
∂t
In addition, the generalized uncertainty relation for operators  and B̂ is
i
∆A ∆ B ≥ -- [ 〈 Â, B̂ 〉 ]
2
which may be re-written as
2 ∆A ∆B ≥ 〈 [ A ˆ , Bˆ ] 〉

6
If operator Bˆ is the Hamiltonian H, then ∆B corresponds to ∆ E so that

〈 Â〉 = h ∆
d
2 ∆A ∆E ≥ 〈 [ Â, H ]〉 = h A
-------
dt ∆t
Since the ∆A terms on the far left and far right of the equation cancel, we have
h
∆ E∆t ≥ ---
2
It might be tempting to make a comparison between this result and the uncertainty rela-
tion for two non-commuting operators such as momentum pˆ x and position xˆ for which
h
∆ p x ∆x ≥ --- follows directly from the generalized uncertainty relation. However, such a
2
comparison would be incorrect since time t is not an operator in quantum mechanics, it
is merely a parameter that advances the system in time. Notice that we could not derive
h
∆ E∆t ≥ --- directly from the generalized uncertainty relation because t is not an opera-
2
tor.

(b) The energy of the n photons oscillating at frequency ω is E = h ω  n +


1
--- so that
 2
∆ E = hω ∆n . The time dependence of the oscillator and its measurable quantities will
be proportional to cos ( ωt – φ ) so that ω ∆t = ∆ φ or ∆t = ∆φ ⁄ ω where ∆φ is the
spread in values of phase φ . From (a) we have
h
∆ E∆t = hω∆ n∆φ ⁄ ω = h ∆n∆φ ≥ ---
2
and hence
1
∆ n∆φ ≥ --
2
For Poisson statistics ∆n = 〈 n〉 so that ∆ φ ≥ 1 ⁄ 4 〈 n 〉 . This situation corresponds
to quasi-classical light in which there are a large number of photons (the classical limit
of large numbers of particles, n) but they have a single frequency of oscillation ω and a
phase coherence that increases with increasing n as 〈 n〉 .

Solution 4
2 2 2
p̂ mω 0 x̂
(a) H = T + V = ------- + --------------- + eE x x̂ cos ( ω t ) .
2m 2

(b) Because the position operator does not explicitly depend on time, 〈 x̂〉 = 0 , we
∂t
have
d i ∂ i –i
〈 x̂ 〉 = - 〈 [ H ,x̂ ]〉 + 〈 x̂ 〉 = -- 〈 [ H ,x̂] 〉 = ---- 〈 [x̂ ,H]〉
dt h ∂t h h
Since [x̂, x̂] = 0 and [ x̂, x̂ 2] = 0 , we are left with

〈 p̂〉
2
d –i p̂ –i p̂ p̂
〈 x̂ 〉 = ---- 〈 [x̂ ,------- ]〉 = ---- 〈 p̂ [x̂, ------- ] + [x̂ ,------- ] p̂〉 = --------
dt h 2m h 2m 2m m

Applied quantum mechanics 7


where we made use of the fact that [ x̂ ,p̂ ] = ih . Notice, that while the position operator
does not explicitly depend on time (i.e., x̂ ≠ x̂ ( t ) ), the expectation value of the position
operator can explicitly depend on time (i.e., 〈 x̂〉 = 〈 x̂〉 ( t ) ).
2
(c) Because [ p̂ ,p̂] = 0 we need only consider the commutator of the momentum oper-
ator with the potential appearing in the Hamiltonian. Hence,
2 2
d 〈 p̂〉 = – i –i mω 0 x̂
---- 〈 [p̂ ,H]〉 = ---- 〈 [ p̂, --------------- + eE x x̂ cos ( ω t ) ] 〉 = – mω 0 〈 x̂〉 – e Ex cos ( ω t )
2

dt h h 2

where we used p̂ = – i h . The spatial derivative of the expectation value of the
∂x
potential is
2mω 0 〈 x̂〉
2
d d
–〈 V ( x̂ )〉 = ---------------------- + e E x cos ( ωt ) = 〈 p̂ 〉
dx 2 dt
which is Ehrenfest’s theorem.
2
d d
Hence, in quantum mechanics, m 〈 x̂ 〉 = – 〈 V ( x̂ ) 〉 . This would be the same as
dt 2 dx
2
dx d
Newton’s second law for force on a particle F = m 2 = – V ( x ) if position
dt dx
d d
x = 〈 x̂ 〉 and if 〈 V ( x̂ ) 〉 = V ( 〈 x̂ 〉 ) . The conditions under which the latter is
dx d 〈 x̂〉
true is when the potential is slowly varying so that higher order terms in the expansion
of the force F ( x̂ ) = F ( 〈 x̂ 〉 ) + ( x̂ – 〈 x̂〉 )F′ ( 〈 x̂ 〉 ) + ( x̂ – 〈 x̂ 〉 ) F′′ ( 〈 x̂〉 ) + … may be
2

neglected so that F ( x̂ ) ≅ F ( 〈 x̂ 〉 ) . This requires that the uncertainty ∆x = ( x̂ – 〈 x̂〉 ) is


small.
d i ∂H ∂H
(d) 〈 H〉 = -- 〈 [H ,H] 〉 + 〈 〉 = 〈 〉 = – eE x ω 〈 x̂〉 sin ( ωt )
dt h ∂t ∂t
where we used the fact that [H ,H] = 0 .
2 2
d d 〈 p̂〉 mω eE
(e) 2
〈 x̂〉 = -------- = – ----------0 〈 x̂〉 – --------x cos ( ω t )
dt d t m m m
this can be written in the familiar form of an undamped forced oscillator
2
d
〈 x̂〉 + mω 0 〈 x̂ 〉 = Fx cos ( ω t )
2
m2
dt
where F x = – e E x . This has an oscillatory solution of the form
F x cos ( ωt )
〈 x̂ 〉 ( t ) = A cos ( ω 0 t – δ ) + --------------------------
-
m ( ω 20 – ω 2 )
which contains two harmonic oscillators, one at the natural frequency ω 0 and the other
at the frequency ω of the input force. The constants A and δ are found from the initial
contitions. If ω 0 = ω the amplitude of the undamped oscillator grows to infinity with
the particular solution increasing linearly in time as

8
Fx t sin ( ω 0 t )
〈 x̂〉 p ( t ) = ---------------------------
-
2mω 0
When ω is close in value to ω 0 then
Fx
〈 x̂〉 ( t ) = --------------------------- ( cos ( ωt ) – cos ( ω 0 t ) )
m ( ω0 – ω )
2 2

which, since 2 sin ( x ) sin ( y ) = cos ( x – y ) – cos ( x + y ) , may be written as


( ω0 + ω ) (ω 0 – ω )
sin  ---------------------t  sin  --------------------- t
2 Fx
〈 x̂〉 ( t ) = ---------------------------
m ( ω0 – ω )
2 2 2 2
The sum frequency ( ω 0 + ω ) is modulated by the difference frequency ( ω 0 – ω ) so
that the amplitude of 〈 x̂ 〉 beats at the difference frequency ( ω 0 – ω ) .

Applied quantum mechanics 9


Applied Quantum Mechanics

Chapter 7 Problems and Solutions

LAST NAME FIRST NAME

Useful constants MKS (SI)

c = 2.99792458 × 10 m s
8 –1
Speed of light in free space
Planck’s constant h = 6.58211889 ( 26 ) × 10 – 16 eV s
h = 1.054571596 ( 82 ) × 10
–34
J s
e = 1.602176462 ( 63 ) × 10
– 19
Electron charge C
m 0 = 9.10938188 ( 72 ) × 10
–31
Electron mass kg
m n = 1.67492716 ( 13 ) × 10
–27
Neutron mass kg
m p = 1.67262158 ( 13 ) × 10
–27
Proton mass kg
k B = 1.3806503 ( 24 ) × 10
–23 –1
Boltzmann constant JK
k B = 8.617342 ( 15 ) × 10
–5 –1
eV K
ε 0 = 8.8541878 × 10
– 12 –1
Permittivity of free space Fm
µ 0 = 4π × 10
–7 –1
Permeability of free space H m
Speed of light in free space c = 1 ⁄ ε0µ0
Avagadro’s number N A = 6.02214199 ( 79 ) × 10 23 m o l –1
Bohr radius a B = 0.52917721 ( 19 ) ×10–10 m
4πε 0 h 2
a B = ----------------
-
m 0e2

Inverse fine-structure constant α –1 = 137.0359976 ( 50 )


4 πε 0 hc
α – 1 = -----------------
-
e2

Applied quantum mechanics 1


PROBLEM 1
(a) Write a computer program to calculate the chemical potential for n non-interact-
ing electrons per unit volume at temperature, T.
(b) Calculate the value of the chemical potential for the case when electrons of
effective mass m = 0.07 × m0 and carrier density n = 1.5 × 10 c m are at tem-
* 18 –3

perature T = 300 K
(c) Repeat (b), only now for the case when electrons have effective electron mass
m = 0.50 × m 0 .
*

(d) Plot the Fermi-Dirac distribution function for the situations described by (b) and
(c).
(e) Repeat (b), (c), and (d), only now for the case when temperature T = 77 K .

Your answer should include a print out of your computer program and plots.

PROBLEM 2
1 1
(a) Show that -------------------------------
( E – µ) ⁄ k B T
= 1 – ------------------------------
(µ – E ) ⁄ kB T
-
e +1 e +1
(b) A semiconductor consists of a valance band with electron energy dispersion relation
EVB = E ( k ) and a conduction band with electron energy dispersion relation such that
EC B = E 0 – E ( k ) , where E0 is a constant such that the conduction band and valence
band are separated by an energy band gap, E g . Show that when particle number is con-
served, the chemical potential is in the middle of the band gap with value µ = E 0 ⁄ 2
and is independent of temperature.

2
SOLUTIONS

Solution 1
(a) and (b) use computer program for Fig. 7.5.
(c) and (d)

Solution 2
(µ – E ) ⁄ k T
B
1 e +1–1 1 1
(a) 1 – ------------------------------
(µ – E ) ⁄ kB T
- = ----------------------------------------
(µ – E ) ⁄ kB T
= ---------------------------------
–( µ – E ) ⁄ kB T
= ------------------------------
( E – µ) ⁄ k B T
-
e +1 e +1 1+e e +1
Because it will be useful for part (b), we modify our notation for the Fermi-Dirac
distribution so that
1
f ( E – µ ) = ------------------------------
( E – µ ) ⁄ kB T
-
e +1
so that
f (E – µ ) = 1 – f( µ – E )
(b) We are given that the semiconductor valance band has an electron dispersion
relation E VB = E ( k ) and a conduction band with dispersion relation such that
E C B = E 0 – E ( k ) , where E0 is a constant such that the conduction band and valence
band are separated by an energy band gap, E g . This means that the conduction and val-
ance bands along with the corresponding density of states will be symmetric around the
middle of the band gap energy that occurs at energy E = E 0 ⁄ 2 .
One way to visualize this is to assume a simple tight-binding band of s-orbitals in
one dimension with lattice spacing L for which E ( k ) = – 2t cos ( k L ) .

Applied quantum mechanics 3


E(k) E(k)

D1 (E0 - E)
E CB = E0 - E(k)

Eg
D1 (E)
E VB = E(k)

0 π/L
Wave vector, k Density of states, D(E)

If the density of states for the valence band is D1 ( E ) , then the total density of states
is D ( E ) = D 1 ( E ) + D 1 ( E 0 – E ) . To calculate the chemical potential we use
∞ ∞

n = ∫ D ( E )f ( E ) d E = ∫ ( D1 ( E ) + D1 ( E 0 – E ) )f ( E ) d E
E=0 E=0
1
where the Fermi-Dirac distribution is f ( E ) = ------------------------------
( E – µ ) ⁄ k BT
- and we note that this has
the property f ( E – µ ) = 1 – f ( µ – E ) e +1
At temperature T = 0 K the integral for the carrier density can be written
∞ EF

n = ∫ D 1 (E )d E = ∫ D 1 ( E ) dE
E=0 E=0

Now, because particle number is conserved, we can equate the T = 0 K integral with
the T ≠ 0 K integral. Using our new notation for the Fermi-Dirac distribution, this is
written
EF

n = ∫ D 1 ( E ) d E = ∫ D 1 ( E ) ( f ( E – µ ) + f ( E0 – E – µ ) ) dE
E=0
which requires
f( E – µ ) + f( E 0 – E – µ ) = 1
f( E – µ ) + ( 1 – f( µ – E 0 + E ) ) = 1
f( E – µ ) – f( µ – E 0 + E ) = 0
– E + µ + µ – E0 + E = 0
E
µ = ----0-
2
This means that the chemical potential is independent of temperature and has a value
that is in the middle of the band gap. This is a result of symmetry built into the density
of electron and hole states in this exercise. If the density of states is not symmetric (as
is usually the case in semiconductors) then the chemical potential is not temperature
independent.

4
Applied Quantum Mechanics

Chapter 8 Problems and Solutions

LAST NAME FIRST NAME

Useful constants MKS (SI)

c = 2.99792458 × 10 m s
8 –1
Speed of light in free space
Planck’s constant h = 6.58211889 ( 26 ) × 10 – 16 eV s
h = 1.054571596 ( 82 ) × 10
–34
J s
e = 1.602176462 ( 63 ) × 10
– 19
Electron charge C
m 0 = 9.10938188 ( 72 ) × 10
–31
Electron mass kg
m n = 1.67492716 ( 13 ) × 10
–27
Neutron mass kg
m p = 1.67262158 ( 13 ) × 10
–27
Proton mass kg
k B = 1.3806503 ( 24 ) × 10
–23 –1
Boltzmann constant JK
k B = 8.617342 ( 15 ) × 10
–5 –1
eV K
ε 0 = 8.8541878 × 10
– 12 –1
Permittivity of free space Fm
µ 0 = 4π × 10
–7 –1
Permeability of free space H m
Speed of light in free space c = 1 ⁄ ε0µ0
Avagadro’s number N A = 6.02214199 ( 79 ) × 10 23 m o l –1
Bohr radius a B = 0.52917721 ( 19 ) ×10–10 m
4πε 0 h 2
a B = ----------------
-
m 0e2

Inverse fine-structure constant α –1 = 137.0359976 ( 50 )


4 πε 0 hc
α – 1 = -----------------
-
e2

Applied quantum mechanics 1


PROBLEM 1
In first-order time-dependent perturbation theory a particle initially in eigenstate | n〉 of
the unperturbed Hamiltonian scatters into state | m〉 with probability a m ( t ) 2 after the
perturbation V is applied.
(a) Show that if the perturbation is applied at time t = 0 then the time dependent coef-
ficient a m ( t ) is
t′ = t
1 iω mn t ′
a m ( t ) = ----- ∫ W mn e d t′
ih t ′ = 0
where the matrix element W mn = 〈m | V |n〉 and hω mn = E m – En is the difference in
eigenenergies of the states |m〉 and | n〉 .
(b) A particle of mass m is initially in the ground state of a one dimensional harmonic
3 –t ⁄ τ
oscillator. At time t = 0 a perturbation V ( x, t ) = V 0 x e is applied where V0 and τ
are constants. Using the result in part (a), calculate the probability of transition to each
excited state of the system in the long time limit, t → ∞ .

PROBLEM 2
An electron is in ground state of a one-dimensional rectangular potential well for which
V ( x ) = 0 in the range 0 < x < L and V ( x ) = ∞ elsewhere. It is decided to control
2 2
–t ⁄ τ
the state of the electron by applying a pulse of electric field E ( t ) = E 0 e ⋅ xˆ at
time t = 0, where τ is a constant, x̂ is the unit vector in the x-direction, and E 0 is the
maximum strength of the applied electric-field.
(a) Calculate the probability P 12 that the particle will be found in the first excited state
in the long time limit, t → ∞ .
(b) If the electron is in a semiconductor and has an effective mass m* = 0.07 × m 0 ,
where m 0 is the bare electron mass, and the potential well is of width L = 10 n m , cal-
culate the value of E 0 for which P 12 = 1 . Comment on your result.

PROBLEM 3
An electron is initially in the ground state of a one-dimensional rectangular potential
well for which V ( x ) = 0 in the range 0 < x < L and V ( x ) = ∞ elsewhere. The
ground state energy is E1 and the first excited state energy is E 2 . At time t = 0 the sys-
2 –t ⁄ τ
tem is subject to a perturbation V ( x , t ) = V0 x e . Calculate and then use a computer
program to plot the probability of finding the particle in the first excited state as a func-
tion of time for t ≥ 0 . In your plot, normalize time to units of τ and consider the three
values of ω 21 = 1 ⁄ 2π , ω 21 = 1 , and ω 21 = 2 π , where h ω 21 = E 2 – E 1 . Explain
your results.

2
PROBLEM 4
4ω e
3 2
〈j |r | k〉 to estimate
2
Use the Einstein spontaneous emission coefficient A = -----------------------
3
3h c 4πε 0
the numerical value of the spontaneous emission lifetime of the 2p excited state of
atomic hydrogen. Use your results to estimate the spontaneous emission lifetime of the
2p transition of atomic He + ions. Describe the spontaneous emission spectral line-
shape you expect to observe. Do you expect the He+ ion 2p spontaneous emission line
spectrum to have a larger or smaller full-width at half maximum compared to atomic
hydrogen?

PROBLEM 5
A particle in a continuum system described by Hamiltonian H( 0 ) is prepared in eigen-
state |n 〉 with eigenvalue E n = hω n . Consider the effect of a perturbation turned on at
time t = 0 that is harmonic in time such that W ( x, t ) = V ( x ) cos ( ωt ) , where V ( x ) is
the spatial part of the potential and ω is the frequency of oscillation. Start by writing
down the Schrödinger equation for the complete system including the perturbation and
then go on to show that the scattering rate in the static limit ( ω → 0 ) is
1 2π
---- = ------ W mn ⋅ D ( E ) δ ( Em – E n ) , where the matrix element W mn = 〈m | W ( x , t ) | n〉
2
τn h
couples state |n〉 to state |m〉 via the static potential V ( x ) , the density of final contin-
uum states is D ( E ) , and δ ( E m – En ) ensures energy conservation.

PROBLEM 6
(a) In a uniform dielectric the dielectric function is a constant over space but depends
on wave vector so that ε = ε ( q ) . Given an impurity potential at position r due to a
2
–e
charge e at position Ri is v q ( r – R i ) = ---------------------- , derive an expression for v ( q ) .
εq r – R i
(b) Use the expression for v ( q ) and Fermi's Golden rule to evaluate the total elastic
scattering rate for an electron of initial energy E(k) due to a single impurity in a dielec-
tric with dielectric function ε = ε ( q ) . Describe any assumptions you have made.
Outline how you might extend your calculations to include elastic scattering from n
ionized impurities in a substitutionally doped crystalline semiconductor.
(c) What differences in scattering rate do you expect in (b) for the case of randomly
positioned impurities and for the case of strongly correlated impurity positions?

Applied quantum mechanics 3


SOLUTIONS
Problem 1
(a) Consider a quantum mechanical system described by Hamiltonian H( 0 ) and for
which we know the solutions to the time-independent Schrödinger equation
H( 0 ) | n〉 = E n | n〉 and the time-dependent Schrödinger equation
∂ – iω nt – iωn t
ih | n〉e = H ( 0 ) |n 〉e .
∂t
At time t = 0 we apply a time-dependent change in potential W(t) so the new Hamil-
tonian is H = H( 0 ) + W ( t ) and the state |ψ ( t ) 〉 evolves in time according to

ih | ψ ( t )〉 = ( H ( 0 ) + W ( t ) ) | ψ ( t )〉
∂t
–i ωn t
where | ψ ( t )〉 = ∑ a n ( t ) |n〉 e and a n ( t ) are time dependent coefficients.
n

Substitution of the state |ψ ( t )〉 in to the time-dependent Schrödinger equation gives


d – iω t – iω t
ih ∑
dt n
a n ( t ) | n〉e n = ( H ( 0) + W ( t ) ) ∑ a n ( t ) |n 〉e n
n

Using the product rule for differentiation ((fg)' = (f 'g + fg')), one may rewrite the left-
hand-side as
∂ ∂
ih ∑   a n ( t ) |n 〉e n + a n ( t )  | n〉e n   = ( H ( 0 ) + W ( t ) ) ∑ a n ( t ) | n〉e n
– iω t – iω t – iω t

n   ∂t   ∂t   n

and remove the term


∂ – iω t – iωn t
ih ∑ a n ( t ) | n〉e n = ∑ a n ( t )H ( 0 ) |n 〉e
n ∂ t n

to leave
–iω n t ∂ – iω nt
ih ∑ | n〉 e a (t) = ∑ a n ( t )W ( t ) | n〉e
n ∂t n n

Multiplying both sides by 〈m | and using the orthonormal relationship 〈 m|n〉 = δ mn


gives
d iω t
ih a m ( t ) = ∑ a n ( t ) 〈m| W ( t ) | n〉 e m n
dt n

However, since
i ωm t – iω nt iωm n t
〈m |W ( t ) | n〉 = ∫ φ *m ( x ) e W φ n ( x )e d x = W mn e

where hω mn = E m – En and W mn is defined as the matrix element ∫ φ *m ( x ) W φ n ( x ) dx ,


we may write
d i ωmn t
ih
dt
am( t ) = ∑ a n( t ) W mn e
n

If the system is initially in an eigenstate | n〉 of the Hamiltonian H ( 0) then


a n ( t = – ∞ ) = 1 and a m ( t = – ∞ ) = 0 for m ≠ n . There is now only one term on the
right-hand-side and we can write
d i ωm nt
ih a m ( t ) = a n ( t ) W mn e
dt

4
Integration from the time when the perturbation is applied at t = 0 gives
t′ = t
1 iω mn t ′
a m ( t ) = ----- ∫ W mn e d t′
ih t ′ = 0

(b) The transition probability from state | n〉 to state |m〉 is P nm = a m ( t ) . Using our
2

solution in part (a) we have


t′ = t 2 2 t′ = t 2
1 iω mn t ′ V ( iωm n –1 ⁄ τ )t ′
P nm = -----2 ∫ 〈m |V ( x, t ) |n〉 e d t′ = -----20 ∫ 〈m |x 3 | n〉 e d t′
h t′ = 0
h t′ = 0

In the problem we have initial state |n = 0〉 and we consider the long time limit,
t → ∞ , this allows us to write
2 t′ = ∞ 2
V0 2 ( imω –1 ⁄ τ )t ′
P nm = -----2 〈m | x |0〉
3
∫ e d t′
h t′ = 0
where, for the harmonic oscillator, the non-zero positive integer m multiplied by the
frequency ω is related to the difference in energy eigenvalue by mω = ( E m – E 0 ) ⁄ h .
The time integral is
t′ = ∞ 2
e ( imω –1 ⁄ τ )t ′ dt′ 1
∫ = ------------------------------
m ω +1⁄τ
2 2 2
t′ = 0
1⁄2
and the matrix elements 〈m |x | 0〉 are found using x = ( h ⁄ 2m ω )

( b̂ + b̂ ) , where
3 *

m * is the particle mass. In our case only transitions |0 〉 → |1 〉 and |0〉 → |3〉 are
allowed.
3⁄2 3⁄2
〈 m|x 3 | 0〉 = ( h ⁄ 2m * ω ) ( b̂ + b̂ b̂ + b̂ ) = ( h ⁄ 2m * ω )
†3 †2 †
( 6δ m = 3 + 3δ m = 1 )
† 1⁄2 1⁄2
where we used b̂ | n〉 = ( n + 1 ) |n + 1〉 , b̂ | n〉 = n | n – 1〉 , b̂ |0 〉 = 0 , and
〈 m| n 〉 = δ mn .
Hence,
t′ = ∞
V2 h -  3  ---------------------------------
2
9 V 20
=  ------------- -2
2 ( imω –1 ⁄ τ )t ′
P 01 = -----20 〈1| x |0〉
3
∫ e d t′
h 2m ω   ( hω ) + h ⁄ τ 
* 2 2
t′ = 0
and
t′ = ∞
h 3
2 2 2
=  ------------- 
V 6V 0
e ( imω –1 ⁄ τ )t ′ d t′
2
P 03 = -----20 〈3| x 3 |0〉 ∫ - -------------------------------------
h  2m * ω   ( 3h ω ) 2 + h 2 ⁄ τ 2 
t′ = 0

So, in the long time limit, t → ∞ , the maximum probability of the transition taking
place occurs as τ → ∞ .

Problem 2
The eigestates of a particle in a one-dimensional rectangular potential well for which
V ( x ) = 0 in the range 0 < x < L and V ( x ) = ∞ elsewhere are
2
| n〉 = -- sin k n x
L
where k n = n π ⁄ L , n = 1, 2 , 3 , … , and eigenenergies are E n = h k n ⁄ 2 m = hω n .
2 2

Applied quantum mechanics 5


2 2
–t ⁄ τ
The perturbation V ( x , t ) = –e x E 0 e is turned on at time t = 0, where τ and E 0
is the maximum strength of the applied electric-field. The probability P 12 that the par-
ticle will be found in the first excited state in the long time limit, t → ∞ is
2 2
t′
2 2 t ′ = ∞ i ω21 t ′ – -----2-
e E0 τ
〈2 |x | 1〉 ∫
2
P12 = ---------------- e dt′
h 2
t′ = 0

where h ω 21 = E 2 – E 1 and
L
2 – 16 L
〈2| x |1 〉 = -- ∫ x sin ( k 2 x ) sin ( k 1 x ) dx = ------------
2
L0 9π
so
2 2
t ′-
t ′ = ∞ i ω21 t ′ – -----
 -----------
16 L  2 2
2 τ

2
P 12 = e E 0 - e dt′
 9hπ 2 
t′ = 0

t ω 21 τ t2 ω 221
Let y = - – i ---------- so that iω 21 t – ----2 = – ------- – y and d t = τ dy . We now can write
2

τ 2 τ 4
t′ = ∞ 2
π e E0 16L 2 –ω221 τ 2 ⁄ 2 2
2 2 2 2
2 16 L 2 –ω 21 τ ⁄ 4
P 12 = e E 0  ------------2  e = -------------------  ------------2  e
2

∫ τ π
2 –y
e dy
9hπ 4 9hπ
t′ = 0

ω 21 = 3 π h ⁄ 2mL
2 2
where and we made use of the standard integral
t′ = ∞
–ax
2
1 π
∫ e dx = -- -- .
2 a
t′ = 0

(b) If the electron is in a semiconductor and has an effective mass m = 0.07 × m 0 ,


*

where m0 is the bare electron mass, and the potential well is of width L = 10 n m , we
can calculate the value of τ for which P 12 is a maximum.
dP 12, t → ∞
0 = = 2τ – ω 221 τ 3

so that τ = 2 ⁄ ω 12 . We now find E 0 for which P 12 = 1 .
2 2
πe 2 E0  16L  2 2π – 1 πe 2 E 0  16L  2  2mL 2 2 – 1
P 12 = ------------------- ------------2 ------- e = ------------------- ------------2 2π ------------
-
 3 π2h  e = 1
4  9 hπ  ω 2 4  9h π 
21

4  9hπ 3π h  1 27h π -  1
2 2 2 2 3 2
------------ -------------2 e = 2  ------------------
2
E0 = -------------
2 2  16 L   e
32 meL 
3
2π e 2mL
– 34 2
27h π 27 × ( 1.05 × 10 ) × π
2 3 3
2  -------------------3  e = 2  -------------------------------------------------------------------------------------------------------
-  × 1.65
0.5
E0 =
 32meL   32 × 0.07 × 9.1 × 10 – 31 × 1.6 × 10 –19 × 10 –24 

E0 = 6.6 × 10 V m
7 –1

This may seem like a large electric field but it corresponds to a voltage drop of only
0.66 V across the potential well of width L = 10 nm.

6
Problem 3
The eigestates of a particle in a one-dimensional rectangular potential well for which
V ( x ) = 0 in the range 0 < x < L and V ( x ) = ∞ elsewhere are

| n〉 = 2
-- sin k n x
L
where k n = n π ⁄ L , n = 1, 2 , 3 , … , and eigenenergies are E n = h k n ⁄ 2 m = hω n .
2 2

The particle is in the ground state prior to a perturbation V ( x , t ) = V0 x 2 e – t ⁄ τ being


turned on at time t = 0. The probability P 12 that the particle will be found in the first
excited state at time t is
iω2 1t ′ – t′
2 t′ = t 2
--
V0 2 τ
〈2| x |1〉 ∫
2
P 12 = ------- e dt′
h 2
t′ = 0

where h ω 21 = E 2 – E 1 and
L
2 – 16 L2
〈 2| x 2 |1〉 = --- ∫ x 2 sin ( k 2 x ) sin ( k 1 x ) dx = --------------
L0 9π 2
The time integral is
– ( 1 ⁄ τ – iω )t
1 + e –2 t ⁄ τ – 2e –t ⁄ τ cos ( ω 21 t )
t′ = t 2 2
– ( 1 ⁄ τ –i ω21 )t ′ 21

∫ e dt′ = e--------------------------------
–1 = ---------------------------------------------------------------
-
t′ = 0 1 ⁄ τ – i ω 21 ω 21 + 1 ⁄ τ
2 2

so
16V 0 L 2 2  1 + e – 2t ⁄ τ – 2e – t ⁄ τ cos ( ω 21 t ) 
P 12 ( t ) =  ----------------
- ---------------------------------------------------------------
-
9h π   ω 21 + 1 ⁄ τ
2 2 2

Now we notice that ω 21 is related to L via ω 21 = 3 π2 h ⁄ 2mL 2 so L 2 = 3π 2 h ⁄ 2 mω 21


and
–2t ⁄ τ –t ⁄ τ
16 V 0  2  1 + e – 2 e cos ( ω 21 t ) 
P12 ( t ) =  ---------------
- ---------------------------------------------------------------
-
 6mω 21   ω 21 + 1 ⁄ τ
2 2 
In the limit t → ∞ the probability becomes
V 0 16  2 
P 12 ( t → ∞ ) =  ---------------
- --------------------------
1
 6mω 21   ω 2 + 1 ⁄ τ 2 
21

In this long time limit and normalizing time to τ , we see that


2
P 12  -- → ∞ , ω 21 = ------  =  -----------
t 1 V 0 16 
- 38.5
τ 2π 6m 
2
P 12  -- → ∞ , ω 21 = 1  =  ------------  0.5
t V 0 16
τ 6m
and
V0 16 2
P 12  -- → ∞ , ω 21 = 2 π =  ------------  6.26 × 10
t –4

τ 6m
so, if we want to plot the results on the same graph, it is best to normalize to these long
time values.

Applied quantum mechanics 7


Problem 4
Differences in energy between eigenfunctions ψ nlm of hydrogen are given by
e4  -1-- – ----
1
- = R y  -----2 – -----2 
m 1 1
∆E = En 2 – En 1 = ------0 -----------------------
2 ( 4 πε 0 ) 2 h 2  n21 n 22 n n 
1 2
where R y = 13.6 eV is the Rydberg energy and ni is the principle quantum number of the
state.

For the 2p state ∆E = hω = -3.401 + 13.606 = 10.205 eV corresponding to a wavelength


λ = c2π h ⁄ ∆E = c 2π ⁄ ω which is λ = 122 nm.
4 πε 0 h 2
An estimate for the matrix element is 〈j |r | k〉 ∼ a B , where a B = ----------------
- is the Bohr
m 0e2
radius and we obtain
( 2π ) --------------- ( 2π ) ∆ E ---------------
3 2 3 3 2
e e 1
⋅ ⋅ a = ----------------------- ⋅ ⋅ a = 1.12 × 10 s = ------
2 2 9 –1
A = -------------
λ
3 3πε 0 h B h c
3 3 3 πε 0 h B τsp
Hence, our estimate of the spontaneous emission lifetime for hydrogen is
τ sp = 0.89 ns .

For the helium ion we note that Z = 2. The Bohr radius scales as a B α 1 ⁄ Z . For a
hydrogenic atom with ion charge number Z the Bohr radius for the electron is
a B = 4πε 0 h ⁄ m 0 Ze 2 . The Rydberg energy which determines ∆E scales as Z2 so
2

∆E = Z 2 R y ( 1 ⁄ n 21 – 1 ⁄ n 22 ) . Hence, the linewidth of the He + ions is


A = A × Z ⁄ Z = 16 × A = 1.79 × 10
* 6 2 10
s –1
and the lifetime is τ
*
sp = 0.056 n s . The

8
Lorentzian line for helium is 16 times wider than that of the corresponding hydrogen
line.

Problem 5
The Hamiltonian H ( 0 ) of the unperturbed system has solutions to the time-independent
Schrödinger equation given by
H ( 0 ) | n〉 = E n | n〉
The time-independent eigenvalues are E n = hω n and the orthonormal eigenfunctions
are |n 〉 . The eigenfunction | n〉 evolves in time according to
–i ωn t –i ωn t
| n〉 e = φn ( x ) e
and satisfies
∂ – iωn t –i ωn t
ih |n 〉e = H ( 0 ) |n 〉e
∂t
At time t = 0 we apply a time-dependent change in potential W(x, t) whose effect is to
create a new Hamiltonian
H = H( 0 ) + W ( x, t )
and state | ψ ( x, t ) 〉 which evolves in time according to

ih |ψ ( x, t ) 〉 = ( H ( 0) + W ( x, t ) ) |ψ ( x , t )〉
∂t
The wavefunction |ψ ( x , t )〉 may be expressed as a sum over the known unperturbed
eigenstates
–i ωn t
| ψ ( x, t ) 〉 = ∑ a n ( t ) |n〉 e
n

where a n ( t ) are time dependent coefficients. It follows that


d – iω t –i ω t
ih ∑
dt n
a n ( t ) | n〉e n = ( H ( 0) + W ( x, t ) ) ∑ a n ( t ) |n〉 e n
n

∂ ∂
ih ∑   a n ( t )  |n 〉e n + a n ( t )  | n〉e n  = ( H( 0 ) + W ( x, t ) ) ∑ a n ( t ) | n〉e n
–i ω t – iω t –iω t

n  ∂ t  ∂ t  n

– iω n t ∂ –i ω n t
ih ∑ | n〉e a (t) = ∑ a n ( t )W ( x, t) |n 〉e
n ∂t n n

Multiplying both sides by 〈 m| and using the orthonormal relationship 〈 m|n〉 = δ mn


gives
d iω t
ih a m ( t ) = ∑ a n ( t ) 〈m |W ( x, t ) |n 〉e m n
dt n

If the perturbation is harmonic and of the form


W ( x, t ) = V ( x ) cos ( ωt )
where the spatial part of the potential is given by V ( x ) . The matrix elements W mn in
the representation of the unperturbed system described by Hamiltonian H ( 0 ) are just
i (ω + ω)t i( ω – ω )t
i ( ωm – ωn )t i( ω m – ωn )t e mn + e mn
W mn e = 〈m | V ( x ) |n〉 e cos ( ω t ) = 〈m |V ( x ) |n〉 ------------------------------------------------
2

Applied quantum mechanics 9


where ω mn = ω m – ω n .
Assuming an initial condition such that a n ( t ≤ 0 ) = 1 for only one eigenvalue and
a m ( t ≤ 0 ) = 0 for m ≠ n , then we may write
t′ = t
1
a m ( t ) = ----- ∫ W mn dt′
ih t ′ = 0
Performing the integration gives
i (ω – ω )t i( ω – ω )t
– 〈m |V ( x ) |n〉 mn mn

a m ( t ) = -----------------------------  e----------------------------- – 1- 
– 1 + e----------------------------
2h  ω mn + ω ω mn – ω 
Taking the static limit ω → 0
i ωm n t
– 〈m |V ( x ) |n〉 e
a m ( t ) = -----------------------------  -------------------- 
–1
h  ω mn 
i ωm n t ⁄ 2 –i ωmn t ⁄ 2
– 〈m |V ( x ) |n〉 i ωm n t ⁄ 2  e
---------------------------------------- 
–e
a m ( t ) = ----------------------------- e
h  ω mn 

– 2 〈 m |V ( x ) |n 〉 iω t ⁄ 2 sin ( ω mn t ⁄ 2 ) 
a m ( t ) = -------------------------------- e m n  ------------------------------
h  ω mn 
Assuming independent scattering channels, the probability of a transition out of state
|n〉 into any state |m〉 is the sum

Pn ( t ) = ∑ am ( t )
2

If there are D ( E ) = dN ⁄ dE states in the energy interval dE = h dω′ , then the sum
can be written as an integral
4 2 dN sin2 ( ( ω′ – ω n ) t ⁄ 2 )
Pn ( t ) = -----2 〈 m |V ( x ) |n 〉 ⋅ ------- ⋅ ∫ ---------------------------------------------- ⋅ hdω ′
h ( ω′ – ω n )
dE 2

To perform the integral we change variables so that x = ( ω ′ – ω n ) ⁄ 2 and then take the
limit t → ∞ . This gives
sin2 ( tx )
------------------ = δ (x )
πt x 2 t→∞

We now note that dE ⁄ d x = 2h since E = h 2x . Hence, the integral can be written


sin 2 ( ( ω – ω n ) t ⁄ 2 ) dE ( tx ) ⋅ tπ hπt sin ( tx )
2 2

∫ -------------------------------------------- ------- dx = 2h ∫ sin


---------------------------- dx = -------- ∫ ------------------ dx
(ω – ω n ) πt4 x πtx
2 2 2
dx 2
so that in the limit t → ∞
sin ( ( ω – ω n ) t ⁄ 2 ) dE hπt
2

∫ -------------------------------------------- ------- dx = -------- ∫ δ ( x ) dx


(ω – ω n )
2 d x 2
One may now write the probability of a transition out of state | n〉 into any state |m〉 as
4 hπt 2π
Pn ( t ) = -----2 〈 m |V ( x ) |n 〉 D ( E ) -------- = ------ W mn D ( E )t
2 2

h 2 h
Recognizing dP n ( t ) ⁄ dt as the inverse probability lifetime τ n of the state |n 〉 , we can
write Fermi’s golden rule

10
1 2π
---- = ------ 〈 m |V ( x ) |n 〉 2 D ( E ) δ ( ω m – ω n )
τn h
The delta function is included to ensure energy conservation. Because we took the
limit ω → 0 the final state must be the same as the initial state energy. If we did not
take the limit ω → 0 so that ω ≠ 0 then the final state energy could be ± hω . The
importance of what we have shown is that Fermi’s golden rule can also be applied to
the static limit of a harmonic perturbation. The significance of the result is that we can
use time dependent perturbation theory to describe scattering from static potentials
such as electrons scattering elastically from ionized impurities in a semiconductor.

Problem 6
(a) and (b) see text.
(c) Here it is important to recognize that there are different limiting cases to con-
sider: (i) Perfectly ordered impurities in a periodic lattice, (ii) clusters of impurity scat-
tering sites and (iii) anticlustering of scattering sites.

Applied quantum mechanics 11


12
Applied Quantum Mechanics

Final

LAST NAME FIRST NAME

SI-MKS

8 –1
Speed of light in free space c = 2.99792458 × 10 m s
– 16
Planck’s constant = = 6.58211889 × 10 eV s
– 34
= = 1.054571596 × 10 Js
– 19
Electron charge e = 1.602176462 × 10 C
– 31
Electron mass m 0 = 9.10938188 × 10 kg
– 27
Neutron mass m n = 1.67492716 × 10 kg
– 27
Proton mass m p = 1.67262158 × 10 kg
– 23 –1
Boltzmann k B = 1.3806503 × 10 JK
–5 –1
constant k B = 8.617342 × 10 eV K
– 12 –1
Permittivity of free space ε 0 = 8.8541878 × 10 Fm
–7 –1
Permeability of free space µ 0 = 4π × 10 Hm
Speed of light in free space c = 1 ⁄ ε0 µ0
23 –1
Avagadro’s number N A = 6.02214199 × 10 mol
– 10
Bohr radius a B = 0.52917721 ×10 m
2
4πε 0 =
a B = ----------------
-
m0e 2

Inverse fine-structure constant α –1 = 137.0359976


4πε 0 =c
α –1 = -----------------
-
e2

Applied quantum mechanics 1


PROBLEM 1
The time-independent Schrödinger equation for Hamiltonian H 0 has known orthonormal eigen-
functions |n〉 such that H 0 |n〉 = E n |n〉 where E n = =ω n . In the presence of a time-dependent
change in potential W(x, t) applied at time t > 0 the system evolves in time according to

i= |ψ ( x, t )〉 = ( H 0 + W ( x, t ) ) |ψ ( x, t )〉 = H |ψ ( x, t )〉
∂t
where
– iω n t
|ψ ( x, t )〉 = ∑ a n ( t ) |n〉e
n

and a n ( t ) are time-dependent coefficients.


(a) Show that for time t > 0
d iω t
i= ----- a m ( t ) = ∑ a n ( t )W mn e mn
dt n

where =ω mn = =ω m – =ω n and W mn = 〈m|W ( x, t ) |n〉 . (30%)


(b) In first-order time-dependent perturbation theory W(x, t) is assumed small and a n ( t > 0 ) is
approximated by its initial value a n ( t = 0 ) . Show that for time t > 0 the final states after scattering
may be found using
t=∞
1 iω mn t′
a m ( t ) = -----
i= ∫ W mn e dt′ (30%)
t=0

(c) If P k → j ( t ) is the probability of a transition from |k〉 at time t′ = 0 to a state |j〉 at time t′ = t
and P j → k ( t ) is the probability that the same Hamiltonian induces a transition from state |j〉 to state
|k〉 , use first-order time-dependent perturbation theory to show P k → j ( t ) = P j → k ( t ) . (40%)

PROBLEM 2
(a) Show that the root-mean-square deviations in the measured value of the observable associated
2 2 1⁄2
with operator  for a system in state ψ n is ∆A = ( 〈 A 〉 – 〈 A〉 ) . (40%)
1⁄2 †
(b) Show that the position operator, x̂ = ( = ⁄ 2 mω ) ( b̂ + b̂ ) and momentum operator
1⁄2 †
p̂ = – i ( =mω ⁄ 2 ) ( b̂ – b̂ ) , for the n-th state of a one-dimensional harmonic oscillator with
1 =
energy eigenvalue E n = =ω  n + --- satisfies the relation ∆x∆p = --- ( 1 + 2n ) . (40%)
 2 2
(c) Calculate the room temperature value of ∆x for a Ge atom (atomic mass 72.6) vibrating as a
harmonic oscillator in one-dimension at a frequency of 9.0 THz. Compare the value you obtain
– 10
with the lattice constant of a germanium crystal ( 5.64613 × 10 m ). (20%)

2
PROBLEM 3
An electron is initially in the ground state of a one-dimensional rectangular potential well for which
V ( x ) = 0 in the range 0 < x < L and V ( x ) = ∞ elsewhere. The ground state energy is E 1 , the
first excited state energy is E 2 , and E 21 = E 2 – E 1 . At time t ≥ 0 the system is subject to a pertur-
–t ⁄ τ
bation V ( x, t ) = – e E 0 xe where τ is a constant and E 0 is the maximum strength of an elec-
tric field applied in the x-direction.
(a) Show that the probability P 12 that the particle will be found in the first excited state as a func-
tion of time for t ≥ 0 is
– 2t ⁄ τ –t ⁄ τ
16e E 0 L 2  1 + e – 2e cos ( ω 21 t )
P 12 ( t ) =  ---------------------
- ---------------------------------------------------------------
- (50%)
 9π 2   2
E 21 + ( = ⁄ τ )
2 
*
(b) The electron in part (a) has an effective mass m = 0.07 × m 0 , where m 0 is the bare electron
mass, the potential well is of width L = 10 nm , and the time constant τ = 1 ps . Calculate the
value of E 0 for which P 12 = 1 in the long time limit t → ∞ . Comment on your result. (50%)

PROBLEM 4
(a) What determines the selection rules for optical transitions at frequency ω between states |k〉 and
|j〉 ? (20%)
(b) Show the inverse of the Einstein spontaneous emission coefficient for a transition from the first
excited state to the ground state
3
1 3πε 0 =c
-2 = τ
--- = -----------------------------
A 2 3
e ω 〈j|r |k〉
can be rewritten for light emission of wavelength λ from electronic transitions in a harmonic oscil-
lator potential as
1 2
--- = 45 × λ ( µm ) = τ ( ns )
A
where wavelength is measured in µm and time τ is measured in ns. You may wish to use the fact
1⁄2 †
that the position operator for a one-dimensional harmonic oscillator is x̂ = ( = ⁄ 2 m 0 ω ) ( b̂ + b̂ ) .
(50%)
(c) Calculate the numerical value of the spontaneous emission lifetime and the spectral linewidth
for an electron mass m0 making a transition from the first excited-state to the ground-state of a har-
–3 –2
monic oscillator potential characterized by force constant κ = 3.59 × 10 kg s . (30%)

Applied quantum mechanics 3


4

Vous aimerez peut-être aussi